teoria combinatoria

59
Dosier módulo VI

description

teoria combinatoria

Transcript of teoria combinatoria

Dosier módulo I

Dosier módulo VI

Módulo VI

61. Unidad I: Principios de Conteo 1.1. ¿QUÉ ES COMBINATORIA Y QUE ESTUDIA?

1.3. PRINCIPIO DE LA MULTIPLICACIÓN 1.4. PRINCIPIO DE CORRESPONDENCIA 1.5. PRICINPIO DE INCLUSIÓN-EXCLUSIÓN 1.6. RECURRENCIA 1.7. PRINCIPIO DE LAS CASILLAS DE DIRICHLET 1.8. PROBLEMAS

2.1. MODELO DE CONJUNTOS 2.2. MODELO DE CAMINOS 2.3. CADENAS DE CEROS Y UNOS 2.4. TRIÁNGULO DE PASCAL 2.5. BINOMIO DE NEWTON 2.6. PROBLEMAS

3. Unidad III: Permutaciones y Arreglos 3.1. PERMUTACIONES 3.2. PERMUTACIONES CIRCULARES 3.3. PERMUTACIONES CON REPETICIÓN 3.4. ARREGLOS 3.5. PROBLEMAS

4. Unidad IV: Extensiones del número combinatorio 4.1. SEPARADORES

1.2. PRINCIPIO DE LA SUMA

2. Unidad II: Combinaciones y el número combinatorio 20

36

43

4.2. MULTICOMBINATORIO4.3. PROBLEMAS4.4. PRINCIPIO DE INCLUSIÓN - EXCLUSIÓN 4.5. DESÓRDENES 4.6. RECURRENCIA 4.7. PRINCIPIO DE CASILLAS4.8. PROBLEMAS

Unamaneramuy sencilla de definir a la Combinatoria es entenderla como aquella área de lamatemá caque trata el problema de contar

.

Esta ac vidad tan natural y a la que nos aproximamos desde edadesmuy tempranas, ene dos caracterís cas centrales:

1. Para contar, hay que considerar todas las posibilidades.

2. Y además, hay que asegurarse que cada objeto de conteo se cuentaexactamente una vez , es decir,hay que evitar contar dos o más veces a un mismo objeto.

Es decir, cada objeto se cuenta al menos una vez ya lo sumo una vez . Sí, esto es evidente, pero prontopodrá darse cuenta que esto puede resultar muy complicado de percibir.

Para contar hay muchas formas, y la más elemental es hacer un conteo exhaus vo, es decir, uno poruno elaborando un listado completo de los objetos, o lo que es lo mismo, un censo. Ahora bien, si loque nos proponemos contar es la can dad de números de un millón de cifras que comienzan con eldígito 4, obviamente, no haremos un censo, nunca terminaríamos; allí entra en juego la combinatoria, lacombinatoria proporciona métodos y técnicas para resolver este y otros problemas en el que el conteoexhaus vo no funciona. Por tal mo vo, algunos consideran a la combinatoria como el arte de ``contarsin contar''.

Principios de conteo

En este apartado desarrollaremos algunos principios básicos de conteo y enumeración. Cuando nosinteresamos en determinar el número de elementos en un conjunto dado, estamos en un caso de conteo;mientras que cuando nos interesa listar los elementos estamos en el caso de enumeración. Ambosproblemas son importantes; hay situaciones en las que nos interesa no sólo saber cuántos elementoshay en un conjunto dado, si no además saber cuáles son tales elementos, de aquí que con frecuencia losmétodos de conteo y enumeración son inseparables. El propósito de este apartado es el de desarrollaralgunas técnicas fundamentales de conteo en los que la enumeración no aparece de manera explícita;se basan fundamentalmente en algunos principios cuya simplicidad con frecuencia impide valorar supotencia; desarrollar la habilidad de aplicar correctamente tales principios requiere alguna prác ca porlo que se proponen ejercicios diversos para la aplicación de los mismos en diferentes contextos.

1.1. ¿QUÉ ES COMBINATORIA Y QUE ESTUDIA?

La Combinatoria es la parte de la Matemá ca que estudia las diversas formas de realizar agrupacionescon los elementos de un conjunto, formándolas y calculando su número. Existen dis ntas formas derealizar estas agrupaciones, según se repitan los elementos o no, según se puedan tomar todos loselementos de que disponemos o no y si influye o no el orden de colocación de los elementos. El desarrollode la combinatoria está fuertemente ligado con su aplicación en la teoría de la probabilidad, pero tambiénes importante en otras ciencias como la informá ca, por ejemplo en la teoría de la codificación y en elanálisis de algoritmos.

El objeto del Análisis Combinatorio oCombinatoria es el estudio de las dis ntas ordenaciones quepuedenformularse con los elementos de un conjunto, de los dis ntos grupos que pueden formarse con aquelloselementos y de las relaciones entre unos y otros grupos

1.2. PRINCIPIO DE LA SUMA

Un concepto fundamental cuando se trata de contar, es el de cardinal de un conjunto. En el caso de losconjuntos finitos, el cardinal de un conjunto A es simplemente el número de elementos que posee y seacostumbra denotarlo por |A| o bien por Card(A).

La técnica más elemental para contar los elementos de un conjunto es la de separar sus elementosen clases disjuntas, de forma tal que su reunión incluya todos los elementos del conjunto. En otraspalabras se requiere que cada elemento del conjunto debe pertenecer a una sola de las clases y quetodo elemento del conjunto pertenece a una de las clases en las que se separa el conjunto.

Definición 1.1. Un conjunto finito A ha sido separado en n clases disjuntas A1, A2, A3, . . . , An, si sesa sface simultáneamente:

. Ai ∩ Aj = ∅ para todo i = j .

.∪ni=1 Ai = A.

7

Tambiéndecimos que el conjuntoAha sidopar cionado y las clases disjuntas (o exlcuyentes) se denominanlos elementos de la par ción.

Teorema 1.1. Principio de la Suma: Si el conjuntoA es posible separarlo en clasesA1, A2, . . . , An, el totalde elementos de A, es igual a la suma de los cardinales de cada una de las clases. Es decir

|A1|+ |A2|+ · · ·+ |An| = |A|

El principio de la suma suele ser enunciado también de la siguiente forma: si un sucesoApuedeocurrir denmaneras, un sucesoB puedeocurrir demmaneras y ambos sucesos nopuedenocurrir simultáneamente,entonces el sucesoA oB puede ocurrir de (n+m) formas. Por supuesto ene su versión cuando haymásdedos sucesos, con la condición obvia quedos cualesquiera de ellos nopuedenocurrir simultáneamente.

1.3. PRINCIPIO DE LA MULTIPLICACIÓN

Cuando se trata de contar parejas (x, y) conociendoel númerodeopciones de cadaunade las componentesdel par, se u liza el conocido como principio de la mul plicación que afirma que el total de posiblespares que se pueden formar es el producto del número de alterna vas que se dispone para la primeracomponente por el número de alterna vas para la segunda componente.

Por ejemplo si se lanza al aire un dado dos veces y anotamos los posibles resultados, estos los podemosregistrarmediante un par (x, y) registrando en la primera componente el resultado de la primera rada yen la segunda componente el resultado de la segunda rada. Siendo que en cada rada hay seis posiblesresultados: 1, 2, 3, 4, 5, 6, el total de posibles pares es 36. En efecto para cada uno de los posiblesresultados para la primera rada tenemos 6 posibles resultados para la segunda rada y siendo quehay seis posibles resultados en la primera rada, el total de resultados será de 36.

De manera completamente análoga suponga que deseamos determinar el total de secuencias de tresletras, es decir ternas (x, y , z) que se pueden formar con las letras a, b, c, d, e, f , de forma tal que no sepermite la repe ción de letras. Habiendo 6 opciones de letras por colocar en la primera posición, para lasegunda posición sólo tendremos 5 opciones puesto que no se acepta la repe ción de letras en la terna;restricción que nos deja en la tercera posición sólo con 4 letras como posibles opciones. Así el total deternas con la restricción planteada será de 6 · 5 · 4 = 120.

Teorema 1.2. Principio del Producto: Si A1, A2, A3, . . . , Ak es una sucesión de conjuntos con cardinalesn1, n2, n3, . . . , nk respec vamente, entonces el conjunto que se ob ene haciendo el producto cartesianode tales conjuntos ene por cardinal el producto de los cardinales de los conjuntos dados. Es decir

|A1 × A2 × · · · × Ak | = n1n2 · · · nk

En par cular, cuando se trata del mismo conjunto A de cardinalidad n en cada uno de los factores, seob ene

|k veces︷ ︸︸ ︷

A× A× · · · × A | = nk

8

En otra versión del principio de la mul plicación, éste se presenta en la forma siguiente: si un conjuntopuede ser par cionado en k clases y cada una de las clases puede ser separada en t pos de elementos,el total de pos de elementos es kt .

Otra versión muy frecuente del principio de la mul plicación se presenta de la forma siguiente: si hayn1 alterna vas de seleccionar un primer objeto y para seleccionar un segundo objeto se dispone de n2formas, la selección del par ordenado de objetos puede ser realizada en n1n2 formas.

Analice el siguiente problema: Si desde Santa Ana a San Salvador se puede viajar por 3 rutas dis ntas deautobus, y de San Salvador a San Miguel se puede viajar por 5 rutas dis ntas de autobus, ¿de cuántasformas dis ntas se puede viajar en autobus desde SantaAna hasta SanMiguel pasandopor San Salvador?¡Inténtelo!

1.4. PRINCIPIO DE CORRESPONDENCIA

Consideremos el problema siguiente: En un campeonato de fútbol se enfrentan n equipos. En cada rondalos equipos perdedores son eliminados. Si en una ronda el número de equipos aún par cipando es impar,uno de los equipos, elegido mediante sorteo, descansa y pasa a la ronda siguiente. ¿Cuántos juegos serealizan durante el campeonato?

Este problema ene por supuestomuchasmaneras de ser abordado, pero en este caso vamos a destacarla forma que se apoya en el denominado principio de correspondencia.

Definición 1.2. Dos conjuntos A y B son tales que a cada elemento de A se le puede asociar uno y sóloun elemento deB y viceversa (a cada elemento deB se le puede asociar uno y sólo uno de los elementosde A), entonces decimos que A y B pueden ponerse en correspondencia biyec va. En este caso se dicetambién que los conjuntos A y B son coordinables y escribimos en forma simbólica A ∼= B.3

Teorema 1.3. Principio de Correspondencia: SiA yB son coordinables entonces enen el mismo númerode elementos o cardinal.

Veamos cómo se aplica el principio anterior al problemapropuesto: Comoen cadapar dohay unperdedory sólo uno y para cada perdedor hay uno y un único par do, para contar el número de par dos nos bastacontar el número de perdedores y siendo que al final del campeonato sólo queda uno de los equipos,el número de perdedores es n − 1, que debe por supuesto ser el número de par dos jugados en elcampeonato. Así, este número debe ser n − 1.

El principio anterior es importante cuando la tarea de contar los elementos de un conjunto A resultamás di cil que la de contar en un conjunto B que puede ponerse en correspondencia biyec va con A.Se cuenta entonces en el conjunto B y automá camente se ha contado en el conjunto A.

He aquí otro ejemplo en donde se aplica el principio de correspondencia biyec va:Queremos determinarel total de formas que tenemos de seleccionar 9 elementos en un conjunto que posee 10 elementos.

3Un conjunto A se dice que es finito si es vacío o si es coordinable con un conjunto de la forma 1, 2, 3, . . . , n y en esteúl mo caso se dice que el cardinal de A es n, o bien que el número de elementos que posee es n. Cuando A es vacío decimosque el conjunto ene cero elementos.

9

Siendo que hay una correspondencia biyec va entre los conjuntos de 9 elementos con los conjuntos de1 elemento, es suficiente para resolver el problema contar el número de selecciones de 1 elemento, queson por supuesto 10 formas.

⋆⋆⋆Analicemos ahora algunas ejemplos unpocomás complicados queu lizan estos tres principios fundamentales;con frecuencia, en un mismo problema se u liza más de uno.

EJEMPLO 1.1Un profesor ene 35 estudiantes en el curso de Álgebra y 38 estudiantes en el curso de Geometría.¿Cuántos estudiantes ene en total?

La respuesta 73 estudiantes sólo sería válida en el caso de que ningún estudiante reciba los dos cursosimpar dos por el profesor. Si hay estudiantes que reciben ambos cursos, el conteo exigiría que se separeel total de estudiantes en las tres clases disjuntas: los que reciben sólo el curso deÁlgebra, los que recibensólo el curso de Geometría y los que reciben ambos cursos. Suponga por ejemplo que hay 10 estudiantesque reciben ambos cursos, entonces los conjuntos no serían disjuntos; si descontamos los estudiantescomunes del curso de Álgebra y Geometría, tendríamos 25 que reciben sólo el curso de Álgebra y 28estudiantes que reciben sólo el curso de Geometría. Así el total es la suma del número de elementosque poseen los tres conjuntos disjuntos, es decir: 25 + 28 + 10 = 63.

EJEMPLO 1.2Se enen6 libros dis ntos deÁlgebra,5 libros dis ntos deGeometría, y4 libros dis ntos de Trigonometría.¿De cuántas formas es posible seleccionar un par no ordenado de libros que no sean de la mismaasignatura?

Siendo que los libros seleccionados deben ser de asignaturas diferentes, las posibilidades de combinarson: Un libro de Álgebra y uno de Geometría; uno de Álgebra y uno de Trigonometría y por úl mo unode Geometría y uno de Trigonometría; con ello hemos separado los pares posibles en casos disjuntos.Aplicando en cada uno de los casos el principio de la mul plicación tenemos que las alterna vas paracada caso son: 6 · 5, 6 · 4 y 5 · 4, respec vamente. Así, por el principio de la suma, el total de alterna vaspara hacer la selección del par de libros es: 30 + 24 + 20 = 74.

EJEMPLO 1.3Se consideran las cadenas de ceros y unos de longitud 5, es decir secuencias con cinco caracteres entrelos dígitos cero y uno. Por ejemplo la cadena 00010 es de longitud cinco. ¿Cuál es el total de cadenas deceros y unos de longitud 5?

Observe que si P = 0, 1 , cada una de las cadenas puede ser iden ficada mediante el quinteto(x1, x2, x3, x4, x5) del producto cartesiano P ×P ×P ×P ×P = P 5, recíprocamente, todo elemento delproducto cartesiano P 5 está asociado a una cadena de ceros y unos de longitud cinco. En consecuencia,dada esta correspondecia biyec va, el total de cadenas será |P 5| = 25 = 32.

10

EJEMPLO 1.4¿Cuál es el número de subconjuntos que posee un conjunto A de cardinal n?

Como en el ejemplo anterior, nos apoyaremos en la correspondencia biyec va entre los subconjuntosde A con las cadenas de longitud n. Supongamos ordenados los elementos de A del primero al n-ésimoelemento y adoptemos la convención siguiente: a cada subconjunto deA asociamos la cadenade longitudn, cuya componente en la posición k-ésimaes0 si el elemento k-ésimodeAnopertenece al subconjunto,o bien 1 si el elemento pertenece al subconjunto. Por ejemplo la cadena (0, 0, 0, 0, . . . , 0) está asociadaal subconjunto vacío; el subconjunto con sólo el primer elementodeA se asocia con la cadena (1, 0, 0, 0, . . . , 0)queposee sólo ceros salvo en la primera componente. Con esta convención se logra en efecto la correspondenciabiyec va y podemos, en vez de contar subconjuntos de A, contar las cadenas de ceros y unos de longitudn, que como extensión del ejemplo anterior será |P n| = 2n (u lizamos el mismo conjunto P del ejemploanterior).

EJEMPLO 1.5Usando las letras a, b, c , d , e, ¿de cuántas formas es posible formar una secuencia ordenada de tresletras en cada uno de los siguientes casos?

a) Si la repe ción de letras está permi da.

b) Si no se permite la repe ción de letras.

c) Sin repe ción de letras y que la secuencia tenga a la letra e.

d) Con repe ción de letras conteniendo e.

a) En el primero de los casos como la repe ción está permi da, el problema es equivalente a determinarel número de ternas del conjunto A3, en donde A = a, b, c, d, e, que es 53 = 125.

b) En el segundo caso no se permite la repe ción, se debe tomar en cuenta que para formar las ternassolicitadas, para la primera componente de la terna disponemos de todos los elementos del conjuntoA; sin embargo para la segunda componente, disponemos de un conjunto con cuatro elementospuesto que debemos excluir el elemento colocado en la primera componente; demanera similar parala tercera componente sólo dispondremos de un conjunto con tres elementos habiendo excluido lospertenecientes a la primera y segunda componente de la terna. Así, el total de ternas con la condiciónsolicitada es: 5 · 4 · 3 = 60.

c) Para el tercer caso, en el que la letra e está obligada a pertenecer a la secuencia, podemos dividir encasos de acuerdo a la posición que ocupe la letra e en la terna, que son obviamente casos disjuntos.Así, si ocupa la primera posición las ternas son de la forma (e, x2, x3) y para la segunda componentese dispone de cuatro elementos en tanto que para la tercera componente se dispone nada más quede tres elementos, por lo que en este caso tendremos 4 · 3 = 12 posibilidades. Los otros casos aconsiderar son las ternas de la forma (x1, e, x3) y las ternas de la forma (x1, x2, e) que son igualmente12 posibilidades en cada uno, por lo que en total tendremos 36 secuencias con la condición dada.

d) Para el úl mo caso en el cual se permite repe ción pero la secuencia con ene obligatoriamente a la e,haremos la separación de ternas en las clases siguientes: (e, x2, x3), (x1, e, x3) y (x1, x2, e), tomando

11

en cuenta que en la primera clase tendremos para x2 y x3 cinco posibilidades, es decir tendremos 25ternas en esta primera clase; en la segunda clase habrá que excluir x1 = e, puesto que las ternascon e en la primera componente ya fueron contadas en las ternas de la primera clase, así, en estasegunda clase de ternas tendremos 4 posibilidades para la primera componente y 5 para la terceracomponente, es decir tendremos 20 ternas en esta clase; en la tercera clase debemos excluir los casosx1 = e y x2 = e puesto que las ternas de este po ya fueron consideradas en las dos clases anterioresy en consecuencia tendremos 4 posibilidades para x1 y 4 posibilidades para x2, resultando 16 ternasde esta clase. En total tendremos 25 + 20 + 16 = 61 ternas. ¡Recuerde que contar significa tomaren cuenta todos los elementos del conjunto pero cada elemento tomado en cuenta sólo una vez!

EJEMPLO 1.6Considere una cuadrícula8×8. Determine el númerode cuadrados formados por vér ces de la cuadrículacuyos lados son paralelos a los segmentos de la cuadrícula.

Este es un pico problema en el que se evidencia la importancia de contar con método, de lo contrariola tarea se vuelve complicada. Tal como lo demanda el principio de la suma, lo mejor es separlos enclases disjuntas. Así, contaremos por separado los cuadrados de lado 1, los cuadrados de lado 2 y asísucesivamente hasta los cuadrados de lado 8.Los cuadrados de lado 1 son 8 por cada fila y siendo que tenemos 8 filas, el total de cuadrados de lado1 es 8 · 8 = 64. Para los cuadrados de lado 2 observe que para formarlos se requiere un par de filasconsecu vas y en ellas los cuadrados de lado 2 que pueden formarse son 7; si se toma en cuenta quese pueden formar siete pares de filas consecu vas se ob ene que el total de cuadrados de lado 2 es7 · 7 = 49. De forma completamente análoga para formar cuadrados de lado 3 requerimos disponer detres filas consecu vas y en ellas es posible construir 6 cuadrados de lado 3; como se pueden formar seistríos de filas consecu vas, el número de cuadrados de lado 3 será de 6 · 6 = 36 cuadrados. Después deconsiderar los casos anteriores los siguientes casos son fáciles de deducir: el total de cuadrados de lado4 será 5 ·5, el de cuadrados de lado 5, será 4 ·4, los de lado 6, serán 3 ·3, los de lado 7 serán 2 ·2 y los delado 8, 1 ·1. Así el total de cuadrados buscado será 1 ·1+2 ·2+3 ·3+4 ·4+5 ·5+6 ·6+7 ·7+8 ·8 = 204cuadrados.

EJEMPLO 1.7Determine el número de diagonales que posee un decágono.

Por supuesto que dibujar las diagonales y después contarlas es una de las opciones para resolver elejercicio, pero obviamente es una solución poco prác ca y di cil de generalizar (y también di cil porqueel dibujo puede complicarse demasiado). Vamos au lizar la separaciónde las diagonales en casos disjuntospara en seguida aplicar el principio de la suma. Recuerde que en un polígono se denomina diagonal alsegmento que une dos vér ces no consecu vos del polígono. Para separar en clases disjuntas vamos aconsiderar las diagonales que “salen” de cada uno de los vér ces. Dado que en un decágono cada vér ceene 7 vér ces no consecu vos son siete diagonales que salen de cada vér ce; como en un decágono

tenemos 10 vér ces, el total de diagonales será de 35 tomando en cuenta que cada diagonal ha sidocontada dos veces, una vez por cada extremo de la diagonal.

12

1.5. PRICINPIO DE INCLUSIÓN-EXCLUSIÓN

Clasificar, como ya se habrá observado, es un método importante cuando se trata de contar objetos.Supone sin embargo que somos capaces de par cionar el conjunto dado en clases disjuntas y esto nosiempre es sencillo. Es necesario desarrollar otros métodos.

Una estrategia que en ciertos casos ofrece alguna ventaja, es la de usar el criterio de contar en elcomplemento, en vez de contar directamente en el conjunto dado. Los dos ejemplos siguientes ilustraneste caso.

EJEMPLO 1.8De las cadenas binarias de ceros y unos, de longitud 10, ¿cuántas enen la propiedad de poseer por lomenos un 0?

EJEMPLO 1.9Cinco niños solicitan, en una sorbetería que dispone de sorbetes de 8 diferentes sabores, un sorbetecada uno. De todas las posibles solicitudes que pueden hacer, ¿en cuántas de ellas los sabores elegidospor los niños, coinciden en cuando menos dos de ellos?

En la solución de los dos problemas anteriores aún cuando es posible clasificar en conjuntos disjuntosde manera directa, el número de clases que resulta es demasiado alto y es preferible contar por sucomplemento.

Hay ocasiones sin embargo en las que la clasificación en el conjunto o en su complemento ofrece elmismo nivel de dificultad y en este caso el principio de inclusión-exclusión resulta ser un valioso recursode conteo.

En su versión más elemental, el principio de inclusión- exclusión nos dice que el número de elementosen la reunión de dos conjuntos A y B, se calcula haciendo la suma de los números de elementos Amaslos de B y se resta el número de elementos comunes a ambos conjuntos, es decir los elementos de laintersección de ambos conjuntos. La interpretación es obvia: la resta del número de elementos de laintersección de los conjuntos evita que éstos se cuenten dos veces por cuanto están incorporados comoelementos del conjunto A y como elementos del conjunto B. En expresión conjun sta, tenemos:

|A ∪ B| = |A|+ |B| − |A ∩ B|

En el caso de la reunión de tres conjuntos A, B, C, los ajustes que debemos realizar para calcular elnúmero de elementos de la reunión de los tres conjuntos sonmenos obvios. En este caso, si sumamos elnúmero de elementos deA,B yC, observamos que los elementos comunes a dos de tales conjuntos hansido contados dos veces, mientras que los elementos comunes a los tres conjuntos han sido contadostres veces. Procedemos a corregir, restando los elementos comunes a dos de los conjuntos dados; eneste caso tenemos tres posibilidades A con B; A con C; B con C. Con esta corrección los elementosestrictamente pertenecientes a dos de los conjuntos dados quedan contados sólo una vez; sin embargo,los elementos comunes a los tres conjuntos, que anteriormente habían sido contados tres veces, ahorahan sido descontados tres veces, lo que hace que en suma no hayan sido contados. Hacemos ahorala corrección final sumando el número de elementos que son comunes a los tres conjuntos. Hemos

13

obtenido en consecuencia el resultado siguiente:

|A ∪ B ∪ C| = |A|+ |B|+ |C| − (|A ∩ B|+ |B ∩ C|+ |C ∩ A|) + |A ∩ B ∩ C|

1.6. RECURRENCIA.

La recursión es un poderoso método de análisis que consiste en suponer que se conocen los valoresde una función de conteo para todos los valores previos a n, e intentar determinar, a par r de talinformación, el valor de tal función para el valor de n.

Cuando se trata de contar objetos en función de una omás variables, la condición ideal es la de lograr unafórmula explícita que relacione tales variables. En ciertas ocasiones, sin embargo, tal condición resultadi cil de alcanzar y con frecuencia es suficiente obtener las suficientes relaciones que nos permitangenerar un algoritmo de cálculo para la resolución de nuestro problema en el caso n-ésimo, a par r dela información acerca de los casos previos, es decir buscar una recurrencia; en otras ocasiones inclusoes preferible y más sencillo buscar tales relaciones. He aquí algunos de ejemplos de cómo esta técnicapermite resolver algunos problemas:

EJEMPLO 1.10Determinar el número de subconjuntos de un conjunto finito dado.

Podemos determinar el número de subconjuntos de un conjunto finito dado a par r de una recurrencia.En efecto, el númerode subconjuntos deun conjunto unitario es 2, y si denotamos el númerode subconjuntosde un conjunto con n elementos comoSn, el número de subconjuntos en el caso de un conjunto con (n+1) elementos puede ser calculadode la siguientemanera: consideremos el úl moelemento incorporado,entonces sólo hay dos posibilidades de subconjuntos, los que no incluyen el elemento úl mo que sonen total Sn, y los que sí lo incluyen, que se ob enen introduciendo éste en cada uno de los subconjuntosde los n primeros elementos, que de nuevo son Sn. Hemos demostrado así que:

Sn+1 = 2Sn

Esta relación junto con la condición inicial S1 = 2, permite calcular todos los valores de Sn, de manerarecurrente, para cualquier número natural n

Sn = 2Sn−1

= 2 (2Sn−2) = 22Sn−2

= 22 (2Sn−3) = 23Sn−3

...

= 2n−1S1 = 2n

EJEMPLO 1.11Determinar el número de diagonales que posee un polígono convexo de n lados.

14

Aún cuando no podamos determinar tal número, podemos intentar establecer una relación entre elnúmero de diagonales de un polígono de n lados con las de un polígono de (n + 1) lados. Sea Dnel número de diagonales de un polígono de n lados. De lo que se trata es de establecer una relaciónentre Dn y Dn+1. Si agregamos un vér ce a un polígono de n lados, éste lo podemos imaginar ubicadoentre dos de los n vér ces an guos. Este vér ce puede ser conectado con n − 2 vér ces an guos paraformar diagonales; así se formarán n−2 nuevas diagonales; sin embargo, un lado en el an guo polígono(justamente el lado que unía los dos vér ces vecinos al nuevo vér ce) se ha transformado en diagonal;en otras palabras el nuevo vér ce ha generado (n − 1) nuevas diagonales. Concluimos entonces quedebe cumplirse la relación de recurrencia:

Dn+1 = Dn + (n − 1)

Por otra parte, observe que D3 = 0. La información anterior es suficiente para calcular el númerode diagonales de cualquier polígono, simplemente aplicando de manera itera va la relación anterior.¡Inténtelo!

1.7. PRINCIPIO DE LAS CASILLAS DE DIRICHLET .

El principio de las casillas de Dirichlet, también conocido como el principio del palomar o principio de loscajones, es un principio fácil de aceptar en su versión más elemental. Su aplicación invade ámbitos muydiversos y con frecuencia su u lización permite resolver problemas que distan mucho de ser triviales.

La idea central del principio de casillas es la siguiente: si se enen n + 1 bolas que serán colocadas enn contenedores (las casillas), entonces forzosamente hay un contenedor que tendrá dos o más bolas,¡lógico! ¿cierto? A con nuación se muestran otras versiones, pero que en escencia son lo mismo

Teorema 1.4. Principio de Casillas:Versión 1: Si más de n bolitas se encuentran distribuidas en n cajas entonces hay una caja que con eneal menos 2 bolitas.Versión 2: Si más demn bolitas se encuentran distribuidas en n cajas entonces hay una caja que con eneal menosm + 1 bolitas.

Versión 3: Si la suma de n can dades es mayor que S entones una de las can dades es mayor queS

n.

Versión 4: Un segmento I de longitud t con ene varios segmentos cuyas longitudes suman más que t ,entonces al menos dos de los segmentos contenidos en I se solapan.Versión 5: Un segmento I de longitud t con ene varios segmentos cuyas longitudes suman más que kt ,donde k es un entero posi vo, entonces hay un punto del segmento I que está contenido en k +1 de lossegmentos contenidos en I.También hay versiones análogas a las úl mas dos para figuras planas y áreas. Versión 6: Sean q1, q2, . . . , qnenteros posi vos. Si q1+q2+ · · ·+qn−n+1 objetos son colocados en n cajas, llamadasC1, C2, . . . , Cn,entonces al menos una caja Ci ene qi objetos o más.

EJEMPLO 1.12De un conjunto de 12 números enteros de dos dígitos, siempre podemos seleccionar dos de ellos cuya

15

diferencia ene la forma aa.4

Obsérvese primero que los números de la forma aa son todos los múl plos de 11 de dos cifras. Ahorabien, si hacemos la división de los números dados por 11, los posibles residuos son 0, 1, 2, . . . , 10, ycomo se escogieron 12 números, hay por lo menos una pareja de números que enen el mismo residuo,entonces su diferencia será divisible por 11 y deberá ser de la forma aa.

Lectura Complementaria: EderAlexander Jacobo.Material deAutoformaciónDocenteparaMatemá ca9° grado. Lección 5. Principios Básicos de Conteo. MINED

1.8. PROBLEMAS

1. Cinco jueces de un deporte determinado disponen de una cartulina en la que por un lado hay un1 y por el otro un 0. ¿Cuántas configuraciones dis ntas pueden generar?

2. Unmarino ene 4 banderas dis ntas para hacer señales. ¿Cuántas señales diferentes puede hacersi coloca 3 banderas en un más l una sobre otra?

3. El ascensor de un edificio de 5 plantas se pone en marcha con 3 pasajeros. ¿De cuantos modosdis ntos se pueden distribuir los pasajeros entre los pisos?

4. Se lanzan dos dados y se observa la suma de los puntos obtenidos. ¿En cuántos casos el resultadoes divisible por tres?

5. En cierto lenguaje de programación un nombre de una variable puede formarse con una letra obien una letra seguida de un un dígito. ¿Cuántas variables diferentes pueden formarse? Asumapara este ejercicio 27 letras.

6. Se ene una ra o banda formada por n rectángulos iguales formando una fila. Cada rectángulopuede colorearse de blanco o negro.

a) ¿Cuántas configuraciones diferentes se pueden hacer?

b) ¿Cuántas formas dis ntas tenemos de colorear la ra, de modo que se obtenga un patrónsimétrico?

c) ¿Y si se desea con colores alternados?

7. ¿De cuantas maneras pueden colocarse un alfil blanco y uno negro en un tablero de ajedrez demodo que se ataquen mutuamente?

8. ¿De cuántas formas es posible seleccionar dos cartas diferentes de una baraja de 52 cartas deforma tal que la primera carta sea un as y la segunda no sea una reina? ¿y si se demanda es que laprimera carta sea de espadas y la segunda no sea reina?

4La notación ab hace referencia a un número cuya cifra de las unidades es a y la cifra de las decenas es b, y la se coloca laraya encima para evitar confundir este número con ab, que representa el producto de a por b. Esta claro que esta notaciónpuede u lizarse para representar números de tres, cuatro o más cifras.

16

9. ¿Cuántos números enteros hay entre 1000 y 10000 con la condición de que sus dígitos seandiferentes? ¿y si se permite la repe ción de dígitos, pero no se permiten los dígitos 2 ó 4? ¿ycuando los dígitos deben ser dis ntos y al menos uno de los dígitos 2 y 4 debe aparecer?

10. Con los dígitos del 0 al 9, ¿cuántas secuencias de longitud 5 pueden ser formadas de forma tal queaparezcan exactamente dos de los diez dígitos?

11. De cuántas formas pueden ser colocadas en un tablero de ajedrez dos torres idén cas en una filacomún o en una columna común? ¿y si las torres no son idén cas?5

12. ¿Cuántos números de cuatro dígitos se pueden formar con los dígitos 1, 2, 3, 4, 5 (con posiblerepe ción) tales que sean divisibles por 4?

13. ¿Cuántos cuadrados de longitud entera pueden formarse en una cuadrícula de n×n (con los ladosformados por segmentos de la cuadrícula)?

14. ¿Cuántas diagonales se pueden trazar en un polígono convexo de n lados?

15. Se dice que en un polígono se ha realizado una triangulación, cuando el interior del polígono secubre con triángulos formados con vér ces del polígono de forma tal que los triángulos no tenganen común más que conjuntos de puntos de área cero. ¿Cuántos triángulos se requieren en unatriangulación en un polígono de n lados?

16. ¿Cuántas secuencias de tres letras diferentes pueden ser formadas haciendo uso de las letrasa, b, c, d, e, f en las cuales aparecen la letra e, o la letra f , o ambas e y f ?

17. Cuando se listan los números del 1 al 10000, ¿cuántas veces se hace uso del dígito 5? ¿y cuántasveces aparece el ``25''?

18. En un tablero de ajedrez, de cuántas formas se pueden colocar dos reinas de forma tal que noestén ni en la misma fila, ni en la misma columna ni en la misma diagonal?6

19. ¿Cuántos números posi vos pueden formarse como suma de los números 1, 3, 5, 10, 20 y 50?

20. Considere una cuadrícula de 5×5 ¿cuántos rectángulos podemos formar con los ver ces de dichacuadrícula cuyos lados sean paralelos a los segmentos de la cuadrícula?

21. Una persona desea invitar a uno o más de sus 10 amigos a su fiesta de cumpleaños. ¿de cuántasformas podría hacer tal invitación?

22. ¿Cuántos números de tres cifras dis ntas pueden formarse con los dígitos impares?

23. Un turista debe trasladarse de una ciudad a otra. Para hacerlo pude optar viajar por avión, autobuso tren, y en cada uno de estos medios puede elegir viajar en 1a clase o en clase turís ca. ¿Decuántas maneras dis ntas puede realizar el viaje?

24. En una fiesta se encuentran 10 hombres y 8mujeres. ¿De cuántas maneras pueden integrarse enparejas para bailar una determinada pieza?

5Cuando un par de torres de ajedrez comparten fila o columna, decimos que las torres se están ``atacando''.6Las reinas no se están ``atacando''.

17

25. Un menú turís co permite seleccionar una entrada de entre cuatro posibles, una comida calientede entre tres, y un postre de entre cinco. ¿De cuántas formas puede elegir su menú un turista sidesea que el pollo guisado y el flan no aparezcan en el mismo menú?

26. ¿Cuántos boletos capicuas de 5 cifras hay?

27. Seis matrimonios posan en fila para una fotogra a. ¿De cuántas maneras pueden ubicarse si losmiembros de cada pareja deben aparecer juntos?

28. Se desea colocar 9 libros dis ntos en tres estantes en una biblioteca, 3 en cada estante. ¿Decuántas pueden ubicarse?

29. Al ingresar a un colegio, 6 chicas debe optar entre asis r a las clases de inglés o a las de francés,¿de cuántasmaneras pueden repar rse si Paula y Daniela deciden elegir lamismamateria y Ceciliano quiere compar r la elección de ellas?

30. Un sábado, cuando iban de compras, Juana y Teresa vieron a dos hombres alejarse en automóvil dela fachada de una joyería, justo antes de que sonará una alarma contra robos. Aunque todo ocurriómuy rápido, cuando fueron interrogadas las dos jóvenes, pudieron dar a la polica la siguienteinformación acerca de la placa (que consta de 2 letras seguidas de 4 dígitos) del automóvil quehuyó: Teresa estaba segura de que la segunda letra de la placa era una O o una Q, y el el úl modígito era un 3 ó un 8. Juana dijo que la primera letra de la placa era una C o unaG y que el primerdígito era defini vamente un 7. ¿Cuántas placas diferentes tendrá que verificar la policía?

31. ¿Cuántos números naturales menores o iguales a un millón no enen dos cifras consecu vasiguales?

32. ¿De cuántas maneras se pueden colocar 8 torres en un tablero de ajedrez tal que no se ataquen?

33. ¿De cuántas maneras se puede escoger un cuadrado negro y uno blanco de un tablero de ajedreztal que no pertenezcan a la misma fila ni a la misma columna?

34. Pruebe que el número máximo de alfiles (todos iguales) que se pueden colocar en un tablerocuadrado de n × n casillas sin que haya dos que se ataquen es 2n − 2, y que el número de estasconfiguraciones es 2n.

35. Entre 20000 y 70000, encuentre el número de enteros pares en los cuales no se repita ningúndígito.

36. Una emisora de radio dispone de seis programas para cubrir las dos horas de mayor audiencia.Dos de estos programas enen una duración de una hora y los cuatro restantes de 30 minutos,siendo todos los programas dis ntos. ¿De cuántas maneras puede la emisora cubrir las dos horasde programación?

37. ¿Cuántos números del 1 al 100000 no son divisibles ni por cinco ni por siete?

38. Tenemos los números del 100000 al 999999. ¿Cuántos de ellos cumplen que enen al menos undígito cero o al menos un dígito uno?

18

39. Dado el conjunto de dígitos 1, 3, 6, 7, 9, determine el número de formas de formar números de4 dígitos tales que sean múl plos de 3.

40. ¿Cuántos divisores posi vos ene el número 1210?

41. María y Laura idean el siguiente juego: cada una lanza un dado, si en los dados sale el mismonúmero, gana Laura; si la suma de ambos es 7, gana María; y en cualquier otro caso hay empate.Calcule las probabilidades de gane de cada una y diga cuál ene mayores chances de éxito.

42. ¿Cuántos cubos diferentes, con sus caras numeradas de 1 al 6 pueden ser fabricados, si la sumade los números que se encuentran sobre cada par de lados opuestos debe ser 7?

43. ¿De cuántas maneras puede colorearse una cuadricula de 8×8 con los colores blanco y negro, detal manera que no existan dos columnas pintadas iguales?

44. ¿De cuántas manereas se pueden colocar las 32 piezas del ajedrez en el tablero sin que los reyesse estén amenazando?

45. Dado el conjunto A = 1, 2, . . . , 10, ¿cuántos cuadrados se pueden formar con los puntos deA2?Nota: los cuadrados pueden ser oblicuos también.

46. Se dibujan n cuerdas no concurrentes en una circunferencia (n ≥ 2). Estas cuerdas se cortanen m puntos al interior de la circunferencia, los cuales subdividen a las cuerdas en r segmentos.Determine r en función de n ym.

Nota: En la figura anterior, se han dibujado n = 4 cuerdas, que se cortan en m = 4 puntos, loscuales subdividen a las cuerdas en r = 12 segmentos.

47. Sea X = 1, 2, 3, . . . , 100 y sea S = (a, b, c) tales que a, b, c ∈ X, a < c y a < c. ¿Cuál esel cardinal de S?

19

1. Aprendizaje inicial de la Lengua es

Combinaciones y el número

combinatorio

20

2.1. MODELO DE CONJUNTOS

Dado un conjunto de n elementos (n ∈ N), ¿cuántos subconjuntos hay de k elementos (0 < k ≤ n)? Aeste número le llamaremos el combinatorio de n escoger k .

Definición 2.1. Número Combinatorio: Dado un conjunto de cardinalidad n, el número de subconjuntosde éste que enen cardinalidad k es el número combinatorio, denotado por

Ckn , (nk) , o bien nCk

Esta claro que este número puede interpretarse como la can dad de formas de escoger k elementos deentre los n disponibles.7

A pesar que de momento no podemos calcular este número, sí podemos hacer un uso habilidoso de éla par r de su definición y de las propiedades que se derivan a par r de ésta.

Es importante mencionar queC0n significa la can dad de subconjuntos de cardinalidad cero que ene unsubconjunto de n elementos, y como el único subconjunto que cumple eso es ∅ entonces C0n = 1 paracualquier natural n; también es fácil argumentar que Cnn = 1 para todo natural n.

EJEMPLO 2.1Suponga que se ene el conjunto A = a, e, i , o, u y se busca la can dad de subconjuntos de A de 3elementos:

a, e, i, a, e, o, a, e, u, a, i , o, a, i , u, a, o, u, e, i , o, e, i , u, e, o, u, i , o, u

En total resultaron 10 por lo que C35 = 10. Observe que para formar un subconjunto de 3 elementosde un conjunto de 5 elementos, pues lo que se hace es escoger cuáles son los tres que conformarán elsubconjunto, y esto significa que al mismo empo se está no escogiendo a los restantes 2, por lo tanto,por cada subconjunto de 3 elementos sí escogidos existe uno y sólo un subconjunto de no escogidos, yviceversa; es decir, por el principio de biyección C35 = C

25 . A con nuación se muestran los subconjuntos

de elementos no escogidos de la sucesión de subconjuntos anteriores:

o, u, i , u, i , o, e, u, e, o, e, i, a, u, a, o, a, i, a, e

Con este razonamiento y sin necesidad de construir todos los subconjuntos, si tenemos un conjunto decardinalidad 9 se tendría que: C09 = C

99 , C

19 = C

89 , C

29 = C

79 , C

39 = C

69 , C

49 = C

59 ¿Observa cuál es el

patrón?

Teorema 2.1. Para todo n ≥ k se cumple la siguiente iden dad combinatoria

Ckn = Cn−kn

7La primera notación u lizada puede tener inver do el orden de los parámetros dependiendo el autor que se consulte.

21

Demostración. Dado que en un conjunto de n elementos, por cada subconjunto de k elementos existeuno y sólo un subconjunto de n − k elementos (una vez escogidos k , los n − k son los no escogidos),y viceversa, entonces contar los subconjuntos de k elementos da el mismo resultado que contar lossubconjuntos de n − k elementos, y de allí se sigue el resultado.

EJEMPLO 2.2Un combinatorio fácil de calcular es C1n , es decir, la can dad de subonjuntos unitarios de un conjuntode n elementos, claramente C1n = n, y por la iden dad anterior también se cumplirá Cn−1n = n.

EJEMPLO 2.3Calculemos C2n .

Se trata por supuesto de elegir una pareja de elementos del conjunto que ene n elementos. Paracalcularlo podemos imaginar ordenados los elementos del conjunto dado. Consideremos primeramentelas parejas que incluyenel primer elemento; éste puede combinarse para formar la pareja con cualesquierade los (n−1) elementos restantes, en consecuencia hay (n−1) parejas que incluyen el primer elemento.En seguida consideremos las parejas que no incluyen el primer elemento pero que incluyen el segundoelemento que son en total (n− 2), ya que estas son las posibilidades de combinar el segundo elementocon cada uno de los elementos restantes. Las parejas que no incluyen ni el primero, ni el segundoelemento, pero sí el tercer elemento, son en número igual (n − 3), y así sucesivamente hasta llegar aconsiderar las parejas que no incluyen a ninguno de los (n−2) primeros elementos, pero sí el penúl mo,que únicamente puede ser combinado con el úl mo, dando así sólo 1 posibilidad. El análisis anterior nospermite afirmar que el númeroC2n es igual a la suma de los números naturales desde el 1 hasta el (n−1),cuyo resultado en representación compacta de sumatoria es:

C2n =

n∑i=1

i =n(n − 1)2

El método de análisis u lizado para calcular C2n , lleva implícito un método para generar las diferentesalterna vas de formar los subconjuntos deseados, problema que con frecuencia es tan omás importanteque saber determinar el número de posibilidades. Sin embargo, cuando de determinar sólo el númerode alterna vas se trata, el método de conteo anterior no resulta tan cómodo y es preciso obtener otraspropiedades que muestran un poco más su lógica interna y que eventualmente simplifican los cálculos.

Por ejemplo, ya se demostró que la can dad de subconjuntos de un conjunto de n elementos es 2n;podemos recalcular este número dividiendo los subconjuntos en clases disjuntas: los subconjuntos decardinalidad0, los subconjuntos de cardinalidad1, los subconjuntos de cardinalidad2, ..., los subconjuntosde cardinalidad n−1, los subconjuntos de cardinalidad n. Dado que la can dad de subconjuntos de cadauna de estas clases me la da un número combinatorio, respec vamente C0n , C

1n , C

2n , . . . , C

n−1n , C

nn , y por

el principio de la suma, se ene demostrado el siguiente teorema:

22

Teorema 2.2. Para todo n se cumple la siguiente iden dad combinatoria

2n = C0n + C1n + C

2n + · · ·+ Cn−1n + Cnn =

n∑k=0

Ckn

Este analisis realmente es fruc fero, podemos demostrar también este otro resultado

Teorema 2.3. Iden dad de Pascal: Para todo n y para todo k ≤ n se cumple la siguiente iden dadcombinatoria

Ckn = Ck−1n−1 + C

kn−1

Demostración. Si deseamos calcularCkn y consideramos un elemento enpar cular, los dos posibles casos(disjuntos) son: que se incluya o no éste elemento en los subconjuntos por formar. Si el elemento seincluye, únicamente hace falta seleccionar (k − 1) elementos de los (n − 1) restantes para completarlos k , y el total de formas de hacer esto es Ck−1n−1 ; si no se incluye hará falta seleccionar los k elementosde los n−1 elementos restantes, es decir, hayCkn−1 posibilidades en este caso. Luego, el teorema quedademostrado por el principio de la suma.

Una de las técnicas básicas de conteo consiste en contar de dosmaneras dis ntas un conjunto de objetos,lo que permite establecer algunas iden dades. He aquí un ejemplo pico:

EJEMPLO 2.4Supongamos que deseamos seleccionar una direc va de k personas en una asamblea de n personas yen la direc va debe tenerse 1 presidente.

Si consideramos el problema de determinar el número de alterna vas que se enen para realizarlo,éste podemos resolverlo en las dos formas siguientes: primero seleccionamos los k direc vos y enseguida seleccionamos el presidente entre los k direc vos; o bien, primero seleccionamos el presidentede entre los n asambleistas y en seguida seleccionamos los k − 1 direc vos restantes de entre los n− 1asambleistas restantes. Al realizarlo de la primera de las formas el número de alterna vas es CknC

1k ; en

la otra forma indicada, se ob ene C1nCk−1n−1 . Así, hemos logrado establecer las relaciones:

CknC1k = C

1nCk−1n−1

kCkn = nCk−1n−1

2.2. MODELO DE CAMINOS

Definición 2.2. Puntos Re culares: Llamaremos puntos re culares o puntos lá ces a puntos (a, b) en elplano cartesiano tales que a, b ∈ Z.

Considere ahora los puntos re culares A(0, 0) yB(k, n− k) ubicados en el primer cuadrante del plano,estos definen una re cula (o malla) tomando estos puntos como esquinas opuestas, y dicha re culaformada por cuadraditos de lado 1. ¿Cuántos caminos sobre la re cula de longitud mínima hay de A a

23

B?

Observe que para que los caminos sean de longitud mínima, los movimientos permi dos sólo son haciala derecha→ y hacia arriba ↑; así, en total, hay que avanzar k unidades hacia la derecha y n − k haciaarriba. Entonces, el problema planteado lo puedo conver r en un problema de decisiones: me muevode punto re cular a punto re cular, hay que avanzar n unidades, avanzando unidad por unidad y a cadapaso decidiendo si es hacia la derecha o hacia arriba; pero k decisiones son obligatoriamente hacia laderecha, y dependiendo cuáles de estas decisiones sean hacia la derecha se me definirá un camino (elresto n−k decisiones forzosamente tendrán que ser hacia arriba), y viceversa. Ahora bien, del conjuntode las n decisiones, la can dad de formas de escoger k para que sean movimientos hacia la derecha esCkn , por lo tanto, el número de caminos buscado es Ckn .

Teorema 2.4. El total de caminos sobre la re cula definida por A(0, 0) y B(k, n − k), y de longitudmínima, es igual a Ckn

..

A(0, 0)

.

B(k, n − k)

.

. . .

.

. ..

. . . ..

...

.

...

Un detalle interesante es que para llegar al punto B(k, n − k) existe únicamente dos formas: o se llegapor el punto re cularB1(k−1, n−k), o bien porB2(k, n−k−1); además, observe que estos casos sonexcluyentes, porque un camino deA aB que pase porB1 yB2 tendrá que tener al menos unmovimientohacia la izquierda o al menos un movimiento hacia abajo, por lo que ya no sería un camino de longitudmínima.

Caso 1: El total de formas de llegar de A a B pasando por B1 (cumpliendo las exigencias, que seancaminos de longitud mínima u lizando únicamente la re cula definida por A yB) es, por el principio dela mul plicación, el total de formas de llegar de A a B1 por el total de formas de llegar de B1 a B. Dadoque para llegar de A aB1 se toman en total n−1 decisiones, de las cuales k −1 son hacia la derecha, eltotal de caminos de longitud mínima de A a B1 es Ck−1n−1 ; además, como el total de formas de llegar deB1 a B u lizando caminos de longitud mínima es 1, se ene que el total de formas de llegar de A a Bpasando por B1 es Ck−1n−1 · 1 = Ck−1n−1 .

Caso 2: Este se resuelve de forma muy similar al caso 1, el total de formas de llegar a A a B pasandopor B2 es igual al producto de la can dad de formas de llegar de A a B2 por la can dad de formas de

24

llegar de B2 a B. Para llegar de A a B2, en total se toman n − 1 decisiones, de las cuales k son hacia laderecha, por lo que el total de caminos de A a B2 es Ckn−1. De aquí se concluye fácilmente que el totalde caminos del caso 2 es Ckn−1.

Finalmente, por el principio de la suma, dado que estos casos son excluyentes, el total de caminos de AaB es igual al total de caminos de A aB pasando porB1 más el total de caminos de A aB pasando porB2, es decir

Ckn = Ck−1n−1 + C

kn−1

Se obtuvo de nuevo la Iden dad de Pascal (2.3). Esta iden dad proporciona un algoritmo (no muyeficiente) para calcular el número combinatorio de forma recursiva, ya que expresa el número combinatoriode subíndice n en términos de combinatorios de subíndices n − 1, que a su vez podrían escribirse entérminos de combinatorios de subíndies n− 2, y así sucesivamente hasta llegar a combinatorios de fácilcálculo. A con nuación mostramos una tabla

k 0 1 2 3 4 5 6 7 8 9

n

0 1

1 1 1

2 1 2 1

3 1 3 3 1

4 1 4 6 4 1

5 1 5 10 10 5 1

6 1 6 15 20 15 6 1

7 1 7 21 35 35 21 7 1

8 1 8 28 56 70 56 28 8 1

9 1 9 36 84 126 126 84 36 9 1

Este modelo del número combinatorio es independiente del modelo de conjuntos, observe que sinmucho esfuerzo pueden hacerse las deducciones C0n = 1 = C

nn , C

1n = n y C

kn = C

n−kn . ¡Inténtelo!

De mayor reto es por ejemplo demostrar por este método el teorema (2.2); este lo podemos analizarde la siguiente forma: considere la siguiente cuadrícula, el total de caminos de longitud n que parten deA(0, 0) formados sólo con movimientos→ y ↑ puede contarse de dos formas dis ntas, la primera porel principio de la mul plicación, como en cada uno de los n movimientos se enen las dos opciones, entotal son 2n caminos; la segunda es dividir estos caminos en casos excluyentes, los caminos que terminanen B0(0, n), los que terminan en B1(1, n − 1), y así sucesivamente, hasta contar los que terminan enBn(n, 0)

25

..

A(0, 0)

.

B0(0, n)

.

B1(1, n − 1)

.

B2(2, n − 2)

.

Bn(n, 0)

Estos conteos devuelven C0n , C1n , . . . , C

nn , respec vamente, y de allí que

2n = C0n + C1n · · ·+ Cnn

2.3. CADENAS DE CEROS Y UNOS

Muchos problemas combinatorios, mediante la biyección adecuada, pueden transformarse a problemasde cadenas de ceros y unos, el número combinatario por sí mismo es un ejemplo de ello.

Teorema 2.5. La can dad de cadenas de k ceros y n − k unos es Ckn .

Demostración. Hasta elmomento se hadichoqueCkn pude interpretarse 1) Como la can dadde subconjuntosde k elementos de un conjunto de cardinalidad n, o lo que es lo mismo, es la can dad de formas deescoger k objetos de entre n objetos diferentes; y 2) representa la can dad de caminos de longitudmínima (sobre la malla) que van del punto A(0, 0) al punto B(k, n − k).

Ahora bien, ambas interpretaciones del número combinatorio pueden asociarse a un tercer problema:¿Cuántas cadenas de ceros y unos se pueden formar con k ceros y n − k unos?

Una cadena de k ceros y n − k unos puede interpretarse de la siguiente forma: Dado el conjunto A =x1, x2, . . . , xn, cada elemento puede o no ser escogido para conformar a un subconjunto; si denotamospor 0 a ``sí se escoge'' y por 1 a ``no se escoge'', una cadena de k ceros y n − k unos representa la síescogitación de k elementos de A y la no escogitación de los restantes n− k , es decir, hace referencia laconformación de un subconjunto de k elementos de A; además, la posición de los ceros hace referenciaa qué elemento de A se escogió; por ejemplo

011 · · · 0︸ ︷︷ ︸n

representa que sí se escoge a x1, que no se escogen x2 y x3,..., que sí escoge xn. El orden entre los cerosy unos puede cambiar, pero la can dad de ceros debe ser k .

Con los caminos, la biyección es aún más evidente: a un movimiento a la derecha (avanza una unidadhorizontalmente) se le asocia un 0, en total son k de estos movimientos, y a un movimiento hacia arriba

26

se le asocia un 1, que son en total n − k ; por tanto, la can dad de cadenas con k ceros y n − k unos esigual a la de caminos de longitud mínima desde A(0, 0) a B(k, n − k), que son en total Ckn .

Finalmente, se puede dar una aproximación al problema de las cadenas de k ceros y n − k unos deforma directa: puedo interpretar esto como que si se enen a disposición n espacios vacíos en los quese colocará o bien un 0 o bien un 1; además, se ene prefijada la can dad de ceros y unos que debencolocarse en esos n espacios, en total son exactamente k ceros y obviamente los restantes n− k debenser unos. Está claro que si se colocan de alguna manera los k ceros, las posiciones que deben ocuparentonces los unos están ya fijadas y no hay nada más que hacer, entonces, me basta conocer de cuántasformas puedo colocar los ceros y luego, las posiciones de los unos quedan determinadas; pero esteconteo es familiar: ¿De cuántas formas puede escoger k espacios vacíos (para colocar los ceros) de losn espacios vacíos disponibles?... pues de Ckn formas.

2.4. TRIÁNGULO DE PASCAL

La siguiente pirámide de números es conocida como Triángulo de Pascal

1

1 1

1 2 1

1 3 3 1

1 4 6 4 1

1 5 10 10 5 1

1 6 15 20 15 6 1

1 7 21 35 35 21 7 1

1 8 28 56 70 56 28 8 1

1 9 36 84 126 126 84 36 9 1...

Cada fila está formada por la anterior, cada número es igual a la suma de los dos números que se ubicanligeramente a la izquierda y a la derecha de la fila anterior, y el número de la primera fila es 1; en casode que sólo tenga uno de esos números, se considera igual a cero al que no aparece en la pirámide.

Teorema 2.6. Los números del triángulo de Pascal son números combinatorios.

Demostración. Dado que el primer número del triángulo es 1, la forma de construcción y la iden dadde Pascal (2.3), el teorema queda demostrado.

27

2.5. BINOMIO DE NEWTON

Considere la expresión

(x + y)n =

n veces︷ ︸︸ ︷(x + y)(x + y) · · · (x + y)

Si se efectúan todos los productos aparecerán los términos xn, xn−1y , xn−2y 2, . . . , xy n−1, y n con susrespec vos coeficientes; entonces la pregunta es ¿cuál es la relación que determina el coeficiente decada término xky n−k? ¿Esta relación dependeden, de k ode ambos?Analicemos algunos casos par culares:

(x + y)0 = 1

(x + y)1 = x + y

(x + y)2 = x2 + 2xy + y 2

(x + y)3 = x3 + 3x2y + 3xy 3 + y 3

(x + y)4 = x4 + 4x3y + 6x2y 2 + 4xy 3 + y 4

(x + y)5 = x5 + 5x4y + 10x3y 2 + 10x2y 3 + 5xy 4 + y 5

...

Si observamos los coeficientes, aparece la siguiente secuencia, ¿la reconoce?

1

1 1

1 2 1

1 3 3 1

1 4 6 4 1

1 5 10 10 5 1...

¡Claro!, vuelven a aparecer los combinatorios, se ob ene el Triángulo de Pascal. Ahora la cues ón esjus ficar por qué los coeficientes del desarrollo (x + y)n son números combinatorios. Lo primero quehay que hacer es entender cómo se ob ene un coeficiente en par cuar al desarrollar (x + y)n. Veamosun ejemplo par cular:

EJEMPLO 2.5Considere (x + y)4 = (x + y)(x + y)(x + y)(x + y), para desarrollarlo, de cada paréntesis debemosescoger la x o la y , y luego semul plican esos cuatro números, al recorrer todas las posibles escogitacionesse ob enen todos los términos, que luego, se suman términos semejantes. Veamos esto en acción:

Se escribirán las escogitaciones así: xxyy representa escoger del primer paréntesis la x , del segundoparéntesis la x , del tercer paréntesis la y , del cuarto paréntesis la y . Entonces, las posibles escogitacionesson, por el principio de la mul plicación, 25 = 32 (dos opciones por cada paréntesis); listadas una a unase ob ene

28

xxxx = x4 xyxx = x3y yxxx = x3y yyxx = x2y 2

xxxy = x3y xyxy = x2y 2 yxxy = x2y 2 yyxy = xy 3

xxyx = x3y xyyx = x2y 2 yxyx = x2y 2 yyyx = xy 3

xxyy = x2y 2 xyyy = xy 3 yxyy = xy 3 yyyy = y 4

Luego, el coeficiente representa la can dadde veces que aparece elmismo término, por ejemplo, observeque el término x2y 2 aparece 6 veces, entonces en el desarrollo de (x + y)4 uno de los sumandos es6x2y 2. Centremos nuestra atención en este término, hay que buscar un método que permita calcularese 6 de forma sistemá ca, quizás esto dé una pista para abordar el caso general:

Para que aparezca el término x2y 2, se necesita claramente un par de x y un par de y , pero una vez quese escoge el par de paréntesis de los cuáles se selecciona la x , está claro que de los otros dos paréntesisdeberá escogerse la y ; así, el total de forma de escogerse 2 paréntesis de los 4 es C24 = 6.

Exactamente los mismo sucede en el caso general, dado (x + y)n, las veces que aparecerá el términoxky n−k es igual a la can dad de formas de escoger k paréntesis de los n (en cada uno de estos paréntesisse seleccionará la x , en los n−k se seleccionará la y ), y esto es Ckn . Así, se ob ene la siguiente iden dadalgebraica llamada el Binomio de Newton:

Teorema 2.7. Binomio de Newton: El desarrollo del binomio (x + y)n es

(x + y)n = Cnnxn + Cn−1n x

n−1y + Cn−2n xn−2y 2 + · · ·+ C1nxy n−1 + C0ny n =

n∑k=0

Ckn xky n−k

Esta iden dad es muy ú l para encontrar y demostrar iden dades combinatorias, observe por ejemplo

EJEMPLO 2.6Si x = y = 1:

2n = (1 + 1)n = Cnn + Cn−1n + Cn−2n + · · ·+ C1n + C0n

Nos encontramos de nuevo con (2.2 ), que ya se derivó a par r de los subconjuntos de un conjunto decardinalidad n, u lizando caminos y ahora con el binomio de Newton. Otro ejemplo

EJEMPLO 2.7Suponga que x = 1, y = −1, se ene entonces

(1− 1)n = Cnn − Cn−1n + Cn−2n − Cn−3n + · · ·+ (−1)n−kCkn + · · ·+ (−1)nC0n∑k par

Ckn =∑k impar

Ckn

Esta es una relación cuya deducciónpor losmétodos anteriores es tremandamente di cil; con la iden dadde Newton se puede ir muy lejos, incluso, es posible evaluar en x o en y valores complejos.

29

Lectura Complementaria: EderAlexander Jacobo.Material deAutoformaciónDocenteparaMatemá ca9° grado. Lección 7. Número Combinatorio. MINED

2.6. PROBLEMAS

1. ¿De cuántas formas pueden 4 niñas y 4 niños ser dividos en dos equipos de 4 si cada equipo debetener al menos a una niña?

2. ¿De cuántas formas pueden 5 niñas y 3 niños ser dividos en dos equipos de 4 si cada equipo debetener al menos a un niño?

3. En una rifa par cipan 100 personas, regalándose 3 televisores idén cos. Quién diseñó la rifo pensóque sacar un cket, luego el segundo y finalmente el tercero era injusto, por lo que se sacan almismo empo los 3 ckets. ¿Cuál es la probabilidad que una persona que compró un solo cketgane?

4. Una asamblea de 14 personas desea elegir entre sus miembros un presidente, un vicepresidentey un secretario de actas. ¿De cuántas formas puede realizarse la elección?

5. Ocho promotores deben visitar 4 comercios. Para ello forman 4 parejas, debiendo cada una deellas visitar un establecimiento. ¿De cuántas formas pueden distribuirse el trabajo?

6. Adrián ene nueve pares dis ntos de calce nes, un día se levantó tarde para su clase de las 9:00AMy en la prisa tomó aleatoreamente ocho calce nes sin mirarlos. ¿Cuál es la probabilidad de queentre los calce nes tomados hayan exactamente dos pares correctos?

7. Se dispone de una colección de 30 pelotas divididas en 5 tamaños dis ntos y 6 colores diferentes,de tal manera que en cada tamaño hay pelotas de todos los colores. ¿Cuántas colecciones de 4pelotas enen exactamente 2 pares de pelotas del mismo tamaño (que no sean las 4 del mismotamaño)?

8. En un curso de Combinatoria hay 30 estudiantes, de los cuales 19 son mujeres. Para organizaruna feria de ciencias, se pide que de este curso se presenten tres exposiciones, realizadas porparejas de estudiantes, tal que cada pareja está intengrada por estudiantes de dis nto género, yun estudiante no puede estar en dos o más parejas. ¿De cuántas formas puede hacerse esto?

9. Se desea elegir una direc va, hay diez candidatos (cinco mujeres y cinco hombres) para los cargosde presidente, vicepresidente, secretario, tesorero y vocal. ¿De cuántas formas pueden elegirselos cargos? ¿De cuántas formas es posible hacerlo si una mujer será la presidenta? ¿De cuántasmaneras es posible si el tesorero ya está definido que será Juan?

10. Una persona ene seis amigos. Cada noche, durante cinco días, invita a cenar a un grupo de tres deellos, de modo que el mismo grupo no sea invitado dos veces. ¿De cuántas formas puede hacerlo?

11. Dados los conjuntos A = 1, 2, . . . , 6 y B = 1, 2, . . . , 10, ¿cuántas funciones estrictamentecrecientes8 f : A→ B pueden definirse? ¿cuántas además cumplen que f (4) = 7?

8Se dice que f es estrictamente creciente si a < b ⇒ f (a) < f (b).30

12. Se dice que una mano de dominó (compuesta por 7 fichas de dominó) ene ``falla'' si alguno delos números entre el 0 y el 6 no aparece en la mano. Determine el número de manos de dominóque no enen falla.

13. En cada subconjunto de 7 elementos del conjunto 1, 2, . . . , 10 se escoje el elemento mayor.¿Cuál es la suma de todos los elementos mayores?

14. Diez puntos están marcados en el plano, no habiendo tres colineales. ¿Cuántos triángulos puedenformarse?

15. ¿Cuántos cuadriláteros (convexos o no convexos) pueden formarse con los vér ces de un n−ágonoregular?

16. Dados seis puntos sobre una circunferencia, se trazan todas las cuerdas que estos puntos definen.¿Cuál es la probabilidad de que al escoger aleatoreamente cuatro de estas cuerdas se forme uncuadrilátero convexo?

17. De cierto número de rectas coplanares se sabe que no hay tres de ellas que concurran en el mismopunto y no hay ninguna pareja de rectas paralelas. Esas rectas producen 45 puntos al cortarse. ¿Decuántas rectas estamos hablando?

18. ¿Cuántos puntos de intersección producen 8 rectas coplanares, sabiendo que dos de ellas sonparalelas y no hay tres concurrentes?

19. Se enennuevepuntos enunplano. Cuatro de ellos están alineados y los restantes estándispuestosde forma que no hay nunca 3 alineados. ¿Cuántos triángulos pueden formarse que tengan susvér ces sobre esos 9 puntos? ¿Cuántas rectas dis ntas determinan esos puntos?

20. En una fábrica hay varios centros de almacenamiento, cada uno de los cuales está unido a losdemás por una cinta transportadora. Calcula el número de centros de la fábrica si se sabe que elnúmero de cintas transportadoras es 66.

21. Dibuja una circunferencia y marca sobre la misma diez puntos. Uniendo parejas de esos puntos¿Cuántos pentágonos convexos dis ntos se podrían formar?

22. Dado el conjunto de dígitos 1, 3, 6, 7, 9, determine el número de maneras de formar númerosde 4 cifras tales que sean múl plos de 3.

23. ¿Cuántos números de cuatro cifras cumplen la propiedad de que el producto de dichas cifras esun cuadrado perfecto?

24. Un cuadrado de lado 6 es dividido en 36 cuadrados de lado 1. Los puntosA yB son puntos mediosde un par de lados opuestos del cuadrado. A cada lado de la línea AB seis cuadrados unitarios sontomados aleatoriamente y coloreados de azul. Si el cuadrado grande es doblado sobre recta AB,calcule la probabilidad de qué exactamente un par de cuadrados azules coincidan.

25. ¿De cuántas maneras puede distribuirse 3n objetos dis ntos en tres cajas dis ntas de modo quecada caja tenga el mismo número de objetos?

31

26. Calcule cuántas cadenas de ceros y unos de longitud 7:

a) con enen exactamente 3 unos y 4 ceros.

b) con enen 4 unos consecu vos.

c) con enen como máximo 3 unos.

d) no con enen 4 o más unos consecu vos.

27. Demuestre la iden dad de Pascal Ckn = Ckn−1 +C

k−1n−1 u lizando dis ntos modelos combinatorios.

28. Demuestre que Ckn = C02Ckn−2 + C

12Ck−1n−2 + C

22Ck−2n−2 .

29. Demuestre Ckn = Ck−1n−1 + C

k−1n−2 + · · ·+ Ck−1k−1 es válido para todo k ≤ n.

30. Demuestre que2n−1∑k=0

Ck2n(−2)k = 0

31. Demuestren∑r=0

3rCrn = 4n

32. Hallar el número de caminos crecientes que empiezan en (0, 0) y terminan en alguno de los puntosindicados.

33. Demuestre por diversos métodos la iden dad de Vandermonde

C0mCrn + C

1mC

r−1n + · · ·+ CrmC0n = Crm+n

34. En la cuadrícula de abajo

..

A(0, 0)

.

B(10, 10)

.

X(6, 4)

a) ¿De cuántas maneras podemos ir de la casilla A a la casilla B con movimientos siempre haciala derecha o bien una casilla hacia arriba?

32

b) ¿De cuántas maneras se puede hacer esto si debemos pasar además por la casilla X?

c) ¿Cuántos caminos llevan de A a B sin pasar por ningún punto de úl ma línea ver cal (salvo Bnaturalmente)

d) ¿De cuántasmaneras se llega deA aB pasandopor un solo puntode la segunda línea horizontal?

35. Demuestre por diversos métodos la siguiente iden dad

Cn2n =

n∑j=0

(C jn)2

36. Dentro de un paralelepípedo rectangular de alambre A de dimensiones 5 × 4 × 6 de colocanalambre dividiendo a A en cubos de lado 1. ¿Cuántos caminos diferentes de longitud mínima haydesde el vér ce inferior izquierdo de la cara anterior de A hasta el vér ce superior derecho de lacara posterior de A?

37. Suponga que ene una armazón cúbica de alambre de l × m × n (la armazón está formada porcubitos de alambre lado 1). Si una hormiga está ubicada en una esquina y quiere llegar caminandopor los alambres a la esquina más alejada con el mínimo recorrido ¿de cuántas maneras puedehacerlo? ¿y de cuántas formas podría hacerlo si la hormiga camina sobre una armazón4−dimensionalde l ×m × n × k? ¿puede generalizar para más dimensiones?

38. Dadounpolígono conexo tal queno ene tres diagonales concurrentes, ¿cuántos puntos de intersecciónal interior del polígono forman las diagonales?

39. Demuestre la iden dad

CqpC0r + C

q−1p−1C

1r+1 + · · ·+ C0p−qC

qr+q = C

qp+r+1

40. Demuestre que CpnCrp = C

rnCp−rn−r siempre que n ≥ p ≥ r .

41. Calculen∑k=1

(k +1

k

)Ckn

42. Demuestre quen∑j=0

C j2n =1

2

(C2n4n + (C

n2n)2)

43. Demuestre las iden dades combinatorias de Chu Shih Chieh

Crr + Crr+1 + · · ·+ Crn = Cr+1n+1

C0r + C1r+1 + · · ·+ Ckr+k = Ckr+k+1

33

44. Demuestre la iden dadk2 = 2C2k + C

1k

y a par r de ella demuestre que

12 + 22 + · · ·+ n2 =1

3n

(n +1

2

)(n + 1)

45. Demostrar que para todo natural n se cumple

Cn+12(n+1) = 2

(2n + 1

n + 1

)Cn2n

46. Resuelva las siguiente ecuaciones combinatorias:

a)C2m−1 + C

2m + C

2m+1 = 19

b)7Cx−12x−2 = 2C

x2x

c)

C0x + C1x · C2x =

x2

2+ 2

d)

C3n − C2n =n3 − 6n2 + 20

6

e)C4n = 13C

2n

f)C12n + C

22n + C

32n = 387n

g)Cn−5n−1 = C

n−7n−3

47. Calcule el valor de C523525 + C524525

48. En la expansión de(20x3

y 5+y 2

x

)67, determinar:

a) El vigésimo término

b) El coeficiente del término cuarenta y cinco

c) El coeficiente de x23, si es posible. Si no, argumente el por qué no existe.

d) ¿Cuántos términos ene la expansión?

49. Encuentre el coeficiente de x5 en la expansión de (1 + x + x2)8 + (1 + x + x2)9.

34

50. En la expansión de(x +

1

x3+ x6 + x7

)5encontrar el coeficiente de x4.

51. Encuentre el coeficiente del término a7b4ce2 en el desarrollo de (a + b + c + d + e)14.

52. ¿Cuál es la suma de todos los números de la forma12!

a!b!c!si a, b, c varían sobre todos los enteros

no nega vos que cumplen que a + b + c = 12?

53. Dada la expresión (7x2 + 12x)25, encuentre la can dad de términos de su expansión, la suma de

los coeficientes de su expansión, el coeficinte de x13 si es posible, y encuentre el i -ésimo término.

54. Demuestre la iden dad del hexágono: Ck−1n−1Ck+1n Ckn+1 = C

kn−1C

k−1n Ck+1n+1 . El nombre viene del

hecho que los coeficientes binomiales implicados forman una figura hexagonal alrededor de Cknen el triángulo de Pascal.

55. Encuentre el coeficiente de xn y de xn+r (con 1 ≤ r ≤ n) en la expansión de

(1 + x)2n + x(1 + x)2n−1 + x2(1 + x)2n−2 + · · ·+ xn(1 + x)n

56. ¿Cuántos paralelogramosquedandeterminados cuandoun grupode 6 rectas paralelas es intersecadopor otro grupo de 6 rectas paralelas?

57. Un tablero de 5× 5 es dividido en 25 cuadrados unitarios, de estos dos son pintados de azul y elresto pintado de blanco, diremos que dos coloraciones son iguales si una puede ser obtenida dela otra al rotar tablero. ¿Cuántas coloraciones dis ntas existen?

58. Un polígono convexo de n lados es tal que no hay punto común alguno para cualesquiera tres desus diagonales. Determine el número de triángulos que se forman de manera tal que dos de susvér ces sean vér ces del polígono y el tercero sea una interseccción de dos diagonales.

59. Una rana se ubica en el tercer escalón de unas gradas, la rana se mueve un escalón por salto.¿Cuántas formas existen para que la rana llegue por primera vez al octavo escalón en su novenosalto?

60. ¿De cuántas formas pueden ordenarse los enteros del 1 al n bajo la siguiente condición: exceptopor el primer entero de la izquierda, todo entero debe diferir por 1 de algún entero que esté másla izquierda que este?

35

2. Aprendizaje de la lectura inicial

Permutaciones y arreglos

36

3.1. PERMUTACIONES

Sea A un conjunto finito. Ordenar elementos de A es darle a cada elemento del conjunto una posicióndeterminada, es decir definir qué elemento ocupa la primera posición, el que ocupa la segunda posición yasí sucesivamente. Dos ordenamientos de elementos deA se dirá que son idén cos si todos los elementosen ambos ordenamientos se encuentran en la misma posición; en consecuencia dos ordenamientosserán diferentes si difieren en la posición en la que se encuentra alguno de los elementos. Hay diversasformas de ordenar los elementos de un conjunto y en este apartado nos ocuparemos de contar el totalde alterna vas de ordenamiento de los elementos de un conjunto de cardinalidad n.

EJEMPLO 3.1¿De cuántas formas pueden ordenarse los elementos del conjunto A = a, b, c?

Los elementos del conjuntoA = a, b, c se puedenordenar de las siguientes seis formas: abc, acb, bac, bca, caby cba. Observe que para contar los posibles ordenamientos nos basta definir el elemento que ocupará laprimera posición, el que ocupará la segunda y el de la tercera posición, lo que es equivalente a contar lasternas (x, y , z) de elementos deA con la condición que los elementos de la terna sean todos diferentes.En este caso, la primera posición puede ser ocupada por uno cualesquiera de los elementos deA, es decircontamos en este caso 3 posibilidades; para la segunda posición ya sólo disponemos de 2 posibilidadesy para la posición tercera sólo hay 1 posibilidad.

En general cuando el conjuntoA ene n elementos, el número total de formas de ordenarlos es n! puestoque la primera posición puede ser ocupada por uno cualesquiera de los n elementos de A; la segundaposición por cualquier elemento que no sea el colocado en la primera posición, por lo que las alterna vasse reducen a (n−1); para la tercera posición se enen (n−2) alterna vas y así sucesivamente dejandoa la úl ma posición con una única alterna va, y por el principio de la mul plicación las alterna vas deordenar los elementos deA seránn(n−1)(n−2) · · · (2)(1) = n!. A cada ordenamiento de los elementosde A se le denomina una permutación, y al total de permutacione se denota por Pn. Tenemos entoncesel resultado siguiente:

Teorema 3.1. El total de permutaciones de n elementos, denotado por Pn es

Pn = n!

3.2. PERMUTACIONES CIRCULARES

Supongamos que debemos colocar a n personas alrededor de unamesa circular en la que se dispone de nposiciones numeradas para su ubicación y deseamosdeterminar el total de alterna vas deordenamiento.Si dos ordenamientos dados los consideramosdiferentes cuando las personas ocupanposiciones numeradasdiferentes, entonces el número de posibles ordenamientos será igual a un ordenamiento en una fila, esdecir, Pn; a estas les llamaremos permutaciones lineales. Sin embargo puede ser que sólo nos interese laposición rela va que guardan entre sí las personas y nonos interese el número de la posición queocupan,a estas permutaciones les llamaremos permutaciones circulares; si este es el caso hay n permutaciones

37

lineales que dejarían a las personas con la misma permutación circular, en efecto, la rotación de laspersonas pasando por las n posiciones numeradas dejan a las personas en la misma posición rela va. Enconsecuencia hayn permutaciones lineales por cadapermutación circular; comoel total de permutacioneslineales es Pn, el total de permutaciones circulares, el cual denotamos por Cn, será

Teorema 3.2. El total de permutaciones circulares de n elementos, denotado por Cn es

Cn =Pnn= (n − 1)!

Hay otra manera de abordar esta mismo problema: Supongamos que una persona X se sienta primero,es indis nto qué asiento escoge dado que lo importante es cómo se ubiquen las restantes n−1 personas,numeramos 1 al asiento que escogió X y a par r de este se numeran los restantes asientos (en ordenhorario o an horario, no importa); ahora las n − 1 personas restantes pueden permutarse linealmentede Pn−1 formas, y una vez que han decidido una permutación lineal en par cular, toman los asientos2, 3, . . . , n en el orden prefijado, eso da nuevamente Cn = Pn−1 = (n − 1)!

Ahora considere un problema parecido pero que entre líneas lleva una restricción más, suponga que seenen 5 objetos dis ntos y se quieren elaborar todos los collares posibles u lizando estos cinco objetos,

¿cuántos hay? ¡Inténtelo!

3.3. PERMUTACIONES CON REPETICIÓN

Suponga ahora un problema dis nto: se quieren permutar n objetos, pero no todos son dis ntos, haydos clases de objetos, hay k1 objetos idén cos de la primera clase y k2 objetos idén cos de la segundaclase (obviamente k1 + k2 = n), ¿cuántas permutaciones hay?

EJEMPLO 3.2¿Cuántas permutaciones se pueden construir con 2 bolas negras idén cas y 2 bolas blancas tambiénidén cas?

Si las 4 bolas fuera todas dis ntas, el total de permutaciones (lineales) sería 4!; enlistemos todas lasposibilidades, se denotará por n1 y n2 a las bolas negras, y por b1 y b2 las blancas:

n1n2b1b2 n1b1n2b2 n1b1b2n2 b1b2n1n2 b1n1b2n2 b1n1n2b2n1n2b2b1 n1b2n2b1 n1b2b1n2 b2b1n1n2 b2n1b1n2 b2n1n2b1n2n1b1b2 n2b1n1b2 n2b1b2n1 b1b2n2n1 b1n2b2n1 b1n2n1b2n2n1b2b1 n2b2n1b1 n2b2b1n1 b2b1n2n1 b2n2b1n1 b2n2n1b1

Ahora, observe que, dado que las dos bolas negras son iguales entre sí, y las dos bolas blancas tambienson iguales entre sí, cada columna, realmente representa a la misma permutación, si les quitamos lossubíndices quedaría

nnbb nbnb nbbn bbnn bnbn bnnb

nnbb nbnb nbbn bbnn bnbn bnnb

nnbb nbnb nbbn bbnn bnbn bnnb

nnbb nbnb nbbn bbnn bnbn bnnb38

Entonces, el total de permutaciones dis ntas bajo estas condiciones no es 4! sino que sólo 6.

¿Cómo podríamos calcular este 6 sin necesidad de hacer una por una todas las permutaciones?, la ideacentral está allí, se trata de, en primer lugar, suponer que todos los objetos son dis ntos, entonces,el total de permutaciones ya se sabe como calcularlo, es 4!; pero, dado que no todos los objetos sondis ntos, hay que ver cuántas veces aparece una verdadera permutación en el conteo anterior; tomepor ejemplo bnnb, cuando se suponen todos dis ntos, las letras b se pueden permutar de 2! formasmientras que las letras n se pueden permutar de otras 2! formas, por lo que bnnb aparecerá 2!2! = 4veces cuando consideremos dis ntos a los objetos (coincide con la tabla, ¿cierto?). Entonces 4! es elcuádruple de las permutaciones que buscamos, es decir, que las permutaciones con objetos repe dos

son en total4!

2!2!= 6.

Resolvamos ahora el caso general: se enen k1 objetos de una clase y k2 objetos de otra clase (los objetosde unamisma clase son idén cos entre sí), y denotamos porP (k1, k2) al total de permutaciones de estosobjetos. Tomando n = k1 + k2, si todos los objetos fueran dis ntos, el total de permutaciones sería Pn,pero en este conteo, cada permutación de las buscadas aparece repe da k1!k2! veces, por lo que

Teorema 3.3. El total de permutaciones con repe ción de k1 objetos de un po y k2 objetos de otro po,denotado por P (k1, k2) y tomando n = k1 + k2 es

P (k1, k2) =n!

k1!k2!

Este argumento se puede generalizar:

Teorema 3.4. Dados k1 objetos idén cos de una clase 1, luego otros k2 objetos idén cos de una clase 2,..., kr objetos idén cos de una clase r, y tomando n = k1 + k2 + · · ·+ kr , el total de permutaciones es

P (k1, k2, . . . , kr) =n!

k1!k2! · · · kr !

Por otra parte, ya antes se había trabajado un problemamuy similar a este, el de las cadenas de longitudn de ceros y unos, todos los ceros son iguales entre sí, todos los unos también, y se está interesado enla can dad de cadenas dis ntas que se pueden formar con k ceros y n − k unos, es decir, todas laspermutaciones con k objetos de idén cos de una clase y n − k de la otra; además, ya se demostró queel total de estas cadenas es Ckn , entonces

Teorema3.5. El número combinatorio puede interpretarse comopermutaciones con elementos repe dos,y se relacionan así

Ckn = P (k, n − k) =n!

k!(n − k)!

Nota: En la relación anterior, observe que pueden darse los casos k = 0 (sólo hay unos) y k = n (sólohay ceros), y según los modelos de número combinatorio, C0n = 1 = C

nn , por lo que para que la relación

anterior siga siendo válida en estos casos, se define 0! = 1.

39

3.4. ARREGLOS

Regresamos a ordenar objetos dis ntos, pero considerando que la can dad de espacios es inferior oigual a la can dad de objetos dis ntos que se ordenan; es decir, ¿cuántas configuraciones de longitud kpuede formarse si se enen n objetos dis ntos (k ≤ n)? A estas configuraciones les llamaremos arreglos,y al total de arreglos de longitud k dados n objetos dis ntos se le denotará por Akn .

9

Este problemaes una aplicación directa del principio de lamu plicación, y sin duda podemos argumentarel caso general sin inconveniente alguno: se enen k espacios, en el primero puede ubicarse a cualquierade los n objetos, hay etonces n opciones; en el segundo espacio puede ubicarse a cualquiera de los nobjetos exceptuando el que se colocó en la primera posición, por lo que hay n − 1 opciones; en eltercer espacio puede colocarse a cualquiera de los n objetos exceptuando el que se colocó en la primeraposición y el que se colocó en la segunda posición, por lo que hay n−2 opciones; este argumento se sigueasí sucesivamente, hasta que en el espacio k puede ubicarse a cualquiera de los n objetos, exceptuandolos que se colocaron en las anteriores k − 1 posiciones, por lo que hay n − (k − 1) opciones. Así

Teorema 3.6. El total de arreglos de longitud k dados n objetos dis ntos (k ≤ n) es

Akn = n(n − 1)(n − 2) · · · (n − (k − 1))

Pero esta expresión puede escribirse de forma más compacta mul plicanndo y dividiendo por (n − k)!

Akn =n(n − 1)(n − 2) · · · (n − (k − 1))(n − k)!

(n − k)! =n!

(n − k)!En par cular si n = k se ob ene

Ann = Pn

Observe queCkn yAkn cuenta estructuras similares, la diferencia radica que cuando se calcula el combinatorio,

únicamente interesa escoger k objetos de los n disponibles (el orden de los objetos no importa), encambio, cuando se calcula la can dadde arreglos, luegode escogerlos se construyen las k!permutaciones(el orden sí importa), entonces

Teorema 3.7. Dados n objetos dis ntos, la can dad de arreglos y de combinaciones de longitud k (k ≤ n)se relacionan por

Akn = k!Ckn

Lectura Complementaria: EderAlexander Jacobo.Material deAutoformaciónDocenteparaMatemá ca9 grado. Lección 6. Permutaciones. MINED

3.5. PROBLEMAS

1. Para confeccionar un examen, se dispone de 3 problemas de Geometría, 4 de Combinatoria y 2de Álgebra. ¿De cuántas maneras pueden ordenarse los problemas si los que corresponde a unmismo tema deben aparecer en forma consecu va?

9La notación más usual para este número es P kn , y a estas se les llama también permutaciones, pero dado que permutarsignifica únicamente cambiar de orden, reservaremos esa palabra exclusivamente para el caso en el que hay igual número deobjetos que de espacios.

40

2. El equipo deAnita y el de Chepito juegan futbol con la regla adicional siguiente: gana quién primeroob ene cuatro goles y no hay empates. ¿De cuántas formas puede ganar el equipo de Anita?

3. ¿De cuántas formas es posible escoger 4 cartas de dis nto manjar de una baraja de 52 cartas?

4. Un bebé recién nacido puede tener 1, 2 ó 3 nombres. ¿De cuántas formas puede llamarse si sepuede escoger de 300 nombres disponibles?

5. En una mesa redonda de 5 asientos se sientan 7 personas. ¿De cuántas formas pueden hacerlo sila persona 1 es enemiga de la persona 2 y si se sienta una no se sienta la otra?

6. ¿De cuántas formas pueden entregarse 6 cartas urgentes si se enen a disposición 3 couriersdis ntos?

7. ¿Cuántas permutaciones hay que no tengan I juntas de la palabra PARANGUATIRIMICUARO?

8. A una conferencia han sido invitadas como exponentes 5 personas: A, B, C, D y E. ¿De cuántasformas se pueden ordenar las exposiciones si B no debe preceder a A? ¿Cuántas formas dis ntasserían si B debe hablar inmediatamente después que A?

9. ¿De cuántas formas pueden sentarse 5 mujeres y 5 hombres en una mesa redonda de tal formaque las personas vecinas de cada quien son de género dis nto?

10. ¿Cuál es la can dad de configuraciones dis ntas que pueden generar los m semáforos ubicadossobre una calle principal?

11. Unamadre ene 2manzanas y 3 peras, y le da a su hija una fruta cada día (de lunes a viernes), ¿decuántas formas puede hacerlo? ¿De cuántas formas podría hacerlo si ene además 4 naranjas?

12. Unaprofesora distribuye5naranjas dis ntas entre sus8 estudiantes de tal formaque cada estudianterecibe a lo sumo una naranja, ¿de cuántas formas puede hacerlo? ¿Cuántas formas serían si ya nose restringe la can dad que puede recibir cada estudiante?

13. Un club depor vo de 30 miembros quiere hacer 4 equipos de 4 personas cada uno para quepar cipen en un rally, ¿de cuántas formas puede hacerlo?

14. ¿De cuántas formas pueden distribuirse m + n + p objetos dis ntos en 3 cajas tal que cada unatengam, n y p objetos respec vamente?

15. Determine el número de permutaciones de la palabra TONACATEPEQUE.

16. Dada una baraja de 4manjares y 52 cartas (diamantes, corazones, espadas y tréboles, 13 cartas decada manjar), ¿de cuántas maneras es posible escoger 4 cartas de dis nto manjar tal que el valorde la carta de diamantes sea igual al valor de la de corazones, y el valor de la carta de espadas seaigual al valor de la de tréboles?

17. Con una baraja igual a la del ejercicio anterior, determine la probabilidad de obtener en la primeramano de un juego de póker: una escalera de color, un póker, una escalera, color, una casa llena,un trío, dos pares.

41

18. Si se juega con 6 dados, dos negros, dos verdes y dos rojos, ¿cuál es la probabilidad de obteneruna escalera tal que los dados del mismo color tengan números consecu vos? (considere al 6 y al1 como consecu vos también)

19. En El Salvador, los números telefónicos se forman con 8 cifras, siendo 2 el primero de ellas, y lasegunda 2, 3, 4, 5 ó 6; en Guatemala en cambio son sólo 7 cifras pero con las mismas restriccionespara sus primeras dos cifras. ¿Cuántas comunicaciones pueden establecerse entre los habitantesde ambos países?

20. ¿Cuántos bloques coloreados diferentes, de forma cúbica fija, puede hacerse si hay seis coloresdisponibles y cada bloque debe tener un color diferente en cada una de sus seis caras?

21. La familia Pérez ha comprado unamesa circular nueva con seis sillas, justa para toda la familia. Paraesta familia, no hay preferencias con respecto a cuál silla ocupar, sólo les interesa saber quienesenen a la par. ¿De cuántas formas pueden sentarse si Juanita y Pedro están peleados y no quieren

sentarse juntos? ¿y si la mamá Pérez quiere sentarse a la par de Anita?

22. Hallar la can dad de números que se obtengan como permutaciones del número 111122256 quesean divisibles por 12.

23. Seis matrimonios se reúnen a cenar en una mesa circular. ¿De cuántas formas pueden ubicarse, sicada hombre debe estar flanqueado por dos mujeres y los miembros de cada pareja deben estarjuntos?

24. Cuatro bailarines y cuatro bailarinas interpretan una danza que consiste en formar una rondatomados de lamano. ¿De cuántas formas puedenubicarse si en la figura deben aparecer alternadamentehombres y mujeres?

25. Sean k, n ∈ N. Demuestre que el número de formas de sentar kn personas alrededor de k mesasdis ntas de modo que hay n personas en cada mesa es

(kn)!

nk

42

4.

Extensiones del número

combinatorio

43

4.1. SEPARADORES

Considere el siguiente problema:

EJEMPLO 4.1Ana quiere comprar 10 dulces para regalárselos a sus primitos; en la enda hay dulces de tres sabores,menta, fresa y limón, ¿de cuántas formas puede escogerlos?

Si llamamosm a la can dad de dulces dementa, f la can dad de dulces de fresa y l la can dad de dulcesde limón, debe cumplirse que m + f + l = 10, y obviamente cada uno de estos números es mayor oigual a cero. Como siempre, lomejor es analizar algunos casos par culares, por ejemplom = 0, entoncesf + l = 10, y las parejas solución son

(f , l) = (0, 10), (1, 9), (2, 8), (3, 7), (4, 6), (5, 5), (6, 4), (7, 3), (8, 2), (9, 1), (10, 0)

en total hay 11 posibilidades. Si analizamos el resto de casos (todos disjuntos) m = 1, 2, . . . , 10, seobtendrán respec vamente 10, 9, . . . , 1 posibilidades, por lo que la can dad de formas que Ana puedehacer la compra es 11 + 10 + · · ·+ 1 = 66.

Este método resolvió el problema y las cuentas no fueron largas ni di ciles, sin embargo no nos da ideade como abordar un problema conmás variables o con númerosmás grandes. Por ejemplo, si los saboresdisponibles fueran 8 y la can dad de dulces que Ana comprará es 100, el problema se vuelve muchísimomás di cil y el método anterior no funcionará bien.

Si se observa, este problema es dis nto a todos los estudiados hasta el momento, y básicamente se tratade buscar combinaciones de objetos, pero no todos los objetos son dis ntos (para el caso, los dulces defresa los consideramos todos iguales, los dementa también y los de limón también). Hay una formamuyingeniosa de resolver este problema, y por lo importante de la técnica, suele dedicársele una sección.

Haremos lo siguiente: los 10 dulces los vamos a interpretar como 10 objetos iguales, 10 bolas porejemplo, y para dis nguir cuáles son de cada sabor, incluiremos 2 ``separadores''; luego, estos 12 objetosse permutan,m es la can dad de bolas que quedan a la izquierda del primer separador, f es la can dadde bolas que quedan entre los separadores y l es la can dad de bolas que quedan a la derecha delsegundo separador. Además, la can dad de permutaciones con repe ción con 10 bolas y 2 separadores

es P (10, 2) =12!

10!2!= 66.

⃝⃝︸︷︷︸m

∣∣∣∣∣∣⃝⃝⃝⃝︸ ︷︷ ︸f

∣∣∣∣∣∣⃝⃝⃝⃝︸ ︷︷ ︸l

El problema general se resuelve de la misma forma:

44

Teorema 4.1. Dada una colección de objetos clasificados en k pos de objetos (los objetos del mismo poson iguales entre sí, y dis ntos de cualquier objeto de otro po), el total de formas de escoger n objetoses

P (n, k − 1) =(n + k − 1)!n!(k − 1)!

⃝|⃝⃝| || ⃝⃝⃝ |⃝⃝ · · ·⃝| ⃝⃝

Demostración. Se considera en principio que los n objetos son todos iguales, y para distribuir los objetosentre las posibles k clases, se agregan k − 1 separadores. El total de configuraciones es igual a laspermutaciones con repe ción P (n, k − 1).

Observe que la respuesta puede verse como un combinatorio también: en total, se enen n + k − 1espacios y se escogen los n (o bien los k − 1) en los que se ubican las bolas (o bien los separadores), porlo que la can dad de configuraciones buscadas es

Cnn+k−1 o bien Ck−1n+k−1

Finalmente, una versión muy u lizada de separadores es la siguiente:

Teorema 4.2. El total de soluciones enteras no nega vas de la ecuación x1+x2+ · · ·+xk = n esCnn+k−1

La relación con el problema anterior es evidente, porque xi representa la can dad de objetos del po i ; elpar cular, el problema de Ana es equivalente a resolver la ecuación x1+x2+x3 = 10, con xi ∈ Z+0 . Noteque hay configuraciones que enen cero bolas de algún po, esto, en el esquema de los separadores, seda cuando los separadores están juntos, o cuando un separador está a la izquierda de todas las bolas oa la derecha de todas las bolas.

Otro detalle importante que comentar es que a veces se busca configuraciones que tengan al menos unode cada po, es decir, xi ≥ 1. En tal caso los separadores se ubican únicamente en los n − 1 espaciosentre las n bolas, a lo sumo un separador por espacio; así, el total de configuraciones con esta nuevarestricción es Ck−1n−1 .

4.2. MULTICOMBINATORIO

Dados n objetos dis ntos, el total de formas de escoger p de ellos es(np

); esto, como ya se estudió en

reiteradas ocasiones, puede interpretarse conmodelo de cajas: en la caja 1 se introducen los p objetos síescogidos, y en la caja 2 los n−p objetos no escogidos, así, este problema es equivalente a: ¿De cuántasformas es posible distribuir n objetos dis ntos en dos cajas, tal que en la primera caja siempre hayanp objetos? Lo interesante de este planteamiento es que se puede extender fácilmente a más númerode cajas; al número que obtendremos en ese caso se le llama mul combinatorio, y no es más que unaextensión y generalización del número combinatorio.

45

Definición 4.1. Mul combinatorio: Al total de formas de distribuir n objetos dis ntos en k cajas, tal queen la primera caja siempre hayan x1 objetos, en la segunda caja x2 objetos, ..., en la k−ésima caja xkobjetos, con x1+x2+ · · ·+xk = n, se le llamamul combinatorio de n escoger x1, x2, . . . , xk , y se denota(

n

x1, x2, . . . , xk

)Es interesante que este problemapuede resolverse sinmayores dificultades a par r del número combinatorioestándar, lo cual se enuncia en el siguiente teorema

Teorema 4.3. El mul combinatorio de n escoger x1, x2, . . . , xk es:(n

x1, x2, . . . , xk

)=

k∏s=1

(n − (x1 + x2 + · · ·+ xs−1)

xs

)Demostración. De los n objetos escogemos los x1 que se colocarán en la primera caja, lo cual se puedehacer de

(nx1

)formas; luego, escogemos los x2 objetos de entre los n − x1 objetos restantes, que se

colocarán en la segunda caja, lo cual es posible de(n−x1x2

)formas, y así sucesivamente, hasta que para la

caja k−ésima se escogerán xk objetos de entre los n − (x1 + x2 + · · · + xk−1) = xk objetos, lo cual sepodrá hacer sólo de una forma,

(xkxk

). Por lo tanto, el total de posibilidades es(

n

x1

)(n − x1x2

)(n − (x1 + x2)

x3

)· · ·(n − (x1 + x2 + · · ·+ xk−1)

xk

)

Este número también ene modelos equivalentes por caminos y por relaciones algebraicas. El vínculodel mul combinatorio con expresiones algebraicas viene de desarrollar los mul nomios, por ejemplo(x+y+z)n, el coeficiente de xay bz c (con a+b+c = n) es el mul combinatorio

(na,b,c

); ¡demuéstrelo!

Además, es importante que iden fique el nexo entre separadores y elmul combinatorio, son problemasaparentemente iguales, pero sonmuydis ntos; observe por ejemplo que el total de términos del desarrollodel trinomio (x + y + z)n es igual a la can dad de términos xay bz c , y esto es igual a la can dad desoluciones de a + b + c = n, lo cual, por separadores es C2n+2.

4.3. PROBLEMAS

1. Determine el númerode formas quepuedenordenarse enunestante4 libros dis ntos deCombinatoria,5 libros dis ntos de Geometría, 3 libros dis ntos de Álgebra y 8 libros dis ntos de Cálculo, si losde Geometría deben estar siempre antes que los de Álgebra.

2. ¿En cuántas de las permutaciones del número 23814425 aparecen los dígitos impares en formacreciente de izquierda a derecha?

3. Se han encargado 20 pupusas de entre los siguientes pos: revueltas, de queso, de chicharrón, defrijol con queso, de queso con loroco y de ayote. ¿De cuántas formas puede hacer la compra?

46

a) Si se ene que llevar al menos 7 de queso.

b) Si se ene que llevar a lo sumo 2 de chicharrón y 10 de ayote.

c) Si se ene que llevar al menos 3 de cada clase.

4. Encuentre el número de secuencias no-decrecientes de largo 10

a1 ≤ a2 ≤ a3 ≤ · · · ≤ a10

donde ai ∈ 1, 2, 3, . . . , 100

5. ?`Cuántas 11-combinaciones pueden formarse de las letras x , y , z si todas las letras deben apareceral menos dos veces y a lo sumo 5?

6. ?`Cuántas cadenas existen de 10 dígitos ternarios (0, 1 ó 2) que contengan exactamente dos 0,tres 1, y cinco 2?

7. ?`Cuántas soluciones existen para la desigualdad

x1 + x2 + x3 ≤ 11

donde x1, x2 y x3 son enteros no nega vos?

8. ?`De cuántas formas pueden ordenarse n ceros y k − 1 unos si no hay dos 1 consecu vos?

9. Se ran 12 dados idén cos al aire, ¿cuál es la probabilidad de que uno de los números 1, 2, . . . , 6no aparezca?

10. ¿De cuántas formas puede distribuirse 20 bolas iguales en 6 cajas, de tal forma que en la primeracaja hay al menos 4 bolas y en la ul ma caja no más de 5?

11. Existen 5 formas de expresar el número 4 como suma de dos enteros no nega vos tomando encuenta el orden: 4 = 0+4 = 1+3 = 2+2 = 3+1 = 4+0. Dados los naturales r y n, determine:

a) El número de formas de expresar 200 en r sumandos.

b) El número de formas de expresar n en 200 sumandos.

c) El número de formas de expresar n en r sumandos tales que todos sean mayores o iguales que5.

12. José comprará 20 galletas de dis ntos sabores: fresa, chocolate, vainilla y limón. ¿De cuántasformas puede hacer la compra si ene que llevar un número de galletas de vainilla que sea 4veces el número de galletas de limón?

13. Determine el número de soluciones enteras no nega vas de 3x1 + 5x2 + x3 + x4 + x5 = 20.

14. Determine el número de términos de la expansión de (x1 + x2 + x3 + · · ·+ xn−1)n−1.

15. Determine el número de soluciones enteras no nega vas de rx1 + x2 + x3 + · · ·+ xn = kr .

16. En la expansión de (a + b + c + d)4847

17. Encuentre el número de enteros posi vos x tales que x ≤ 9999999 y la suma de sus dígitos seaigual a 31.

18. Si Lorena ra un dado cinco veces, ¿cuál es la probabilidad de que la sumas de sus cinco radassea 20?

19. ¿Cuántas soluciones hay, entre 1 y 9 inclusive, de la ecuación x1 + x2 + x3 + x4 = 26?

20. Encuentre el número de soluciones enteras posi vas de la ecuación:

(x1 + x2 + x3)(y1 + y2 + y3 + y4) = 77

a) Encontrar el coeficiente de a8b10c15d15.

b) Determinar el número de términos de la expansión.

21. Determine el coeficiente de x5 en la expansión de (1 + x + x2 + · · ·+ x1000)6.

4.4. PRINCIPIO DE INCLUSIÓN - EXCLUSIÓN

El principio, puede ser extendido extendido de forma natural al caso de la reunión de n conjuntos. Laforma que adopta el principio en este caso es:

Teorema 4.4. Principio de Inclusión - Exclusión: Dados n conjuntosA1, A2, . . . , An, el cardinal de la uniónestá dado por

∣∣∣∣∣n∪i=1

Ai

∣∣∣∣∣ =∑i

|Ai | −∑i<j

|Ai ∩ Aj |+∑i<j<k

|Ai ∩ Aj ∩ Ak |+ · · ·+ (−1)n+1|A1 ∩ A2 ∩ · · · ∩ An|

La fórmula pretende asegurar que todos los elementos son contados una y sólo una vez, lo que puedeser verificado después de un momento de reflexión.

4.5. DESÓRDENES

EJEMPLO 4.2Si consideramos el orden de los números naturales 1, 2, 3, 4, en la permutación 3142 ningún elementoestá en su posición natural. A una permutación con tal propiedad le denominaremos un desorden o undesarreglo. ¿De los 24 posibles ordenamientos de tales números, cuántos desórdenes existen?

Denotemos por D4 tal número de desórdenes; por F1 el número de las permutaciones que dejan fijoun elemento en su posición; por F2 las que dejan en su posición dos de los 4 elementos, por F3 las quedejan en su puesto tres de los 4 elementos; de manera completamente similar definiremos F4. Así, porel principio de inclusión-exclusión (4.4) tenemos el siguiente resultado:

D4 = 4!− F1 + F2 − F3 + F448

Ahora bien, F1 se descompone en los que dejan fijo el 1 en su posición, que son 6, los que dejan fijo el2, que son otras 6; hay 6 que dejan el 3 en su posición y 6 que dejan fijo el 4; así, F1 es 24. De las quedejan fijos dos de los cuatro elementos están los que dejan fijos el 1 y 2, los que dejan fijos el 1 y 3, el 1y 4; los que dejan fijos el 2 y 3, el 2 y 4; finalmente tenemos los que dejan fijos el 3 y el 4. Como en cadauno de estos caso, que son seis, el número de permutaciones es 2, resulta que F2 es 12. El caso de quedejen fijos tres elementos con ene los casos siguientes: que dejen fijos 1, 2, 3; 1, 2, 4; 1, 3, 4 y el casoque dejen fijos 2, 3, 4; en total son cuatro casos y cada uno de ellos ene una permutación por lo queF3 es igual a 4. Finalmente, F4 con ene una única permutación, es decir F4 es 1. En resumen tenemos:

D4 = 24− 24 + 12− 4 + 1 = 9Hay en consecuencia 9 desórdenes en las permutaciones de orden 4.

Esta claro que esta relación se puede generalizar.

Teorema 4.5. El total de desórdenes de orden n, denotado porDn, es

Dn =

n∑k=0

(−1)k(n

k

)(n − k)!

Demostración. El total de permutaciones es Pn = n!. Tomando la misma notación del ejemplo anterior,Fk denota aquellas permutaciones que enen a k (al menos) de sus elementos en la posición que lescorresponde; así, el total de depermutaciones deFk lo contamosprimero escogiendo los k quequedaránen la posición que les corresponde, lo cuál es posible hacerlo de Ckn formas, y luego permutando losrestantes n − k objetos, lo cual es posible hacerlo de Pn−k = (n − k)! formas, y por el principio de lamul plicación, Fk = Ckn (n − k)!. Luego, por el principio de inclusión-exclusión:

Dn = n!− F1 + F2 − F3 + · · ·+ (−1)kFk + · · ·+ (−1)nFn= (−1)0C0n(n − 0)! + (−1)1C1n(n − 1)! + (−1)2C2n(n − 2)! + · · ·+ (−1)nCnn(n − n)!

=

n∑k=0

(−1)k(n

k

)(n − k)!

Observe que esa expresión se puede manipular algebraicamente, y reescribirse como

Dn = n!

n∑k=0

(−1)k

k!

4.6. RECURRENCIA

EJEMPLO 4.3La Torre de Hanoi:Se dispone de n discos de diferentes medidas y de tres clavijas en donde éstos pueden ser colocados.Inicialmente todos los discos están colocados en una de las clavijas y ordenados de abajo hacia arriba demayor a menor. El problema consiste en trasladarlos a otra de las clavijas siguiendo las reglas siguientes:

49

) Los discos se mueven uno por uno.

) En ningún caso puede colocarse un disco sobre un disco de radio menor.

Obsérvese que para cumplir con la condición segunda resulta indispensable disponer de tres clavijas.El problema consiste en determinar el número mínimo de movimientos requeridos para pasar todos losdiscos de una clavija a otra.

Sea mn el número mínimo de movimientos para pasar n discos de una clavija a otra. Es obvio que parapoder mover el disco que se encuentra en el fondo, debemos pasar de una clavija a otra los n− 1 discosmenores a otra clavija, lo que requieremn−1 como número mínimo de movimientos. Hecho lo anterior,debemos pasar el disco mayor a la tercera clavija, lo que exige un movimiento, y finalmente pasar losn−1 discosmenores a esta úl ma clavija, lo que por definición requiere nuevamentemn−1movimientoscomo mínimo. Así en total se requieren como mínimo mn = 2mn−1 + 1 movimientos. Por otra parte,cuando se ene sólo 1 disco, obviamente, el número mínimo de movimientos esm1 = 1; es decir:

mn = 2mn−1 + 1

m1 = 1

Información suficiente para calcular el valor demn para los diferentes valores de n.

EJEMPLO 4.4La Sucesión de Fibonacci:

Suponemos que cada mes la hembra de una pareja de conejos pare una pareja de conejos (de diferentesexo). Dos meses más tarde la hembra de la nueva pareja pare una nueva pareja. Determinar el númerode parejas de conejos al finalizar el primer año, si se supone que al inicio sólo se dispone de una parejade conejos en edad reproduc va y que en el período en mención no hay defunciones.

Por ejemplo, al final del primer mes tendremos dos parejas, al final del segundo mes se tendrán tresparejas. Denotemos por Fn el número de parejas al final del enésimo mes. Este número de parejas debeser igual al número de parejas en el mes n−1, que es igual a Fn−1, más los recién nacidos, cuyos padresúnicamente pueden ser las parejas existentes en el mes n − 2. Así, resulta la relación:

Fn = Fn−1 + Fn−2F0 = 1

F1 = 2

De nuevo, con estas relaciones podemos calcular el valor de Fn para los diferentes valores de n. Conella se logra la conocida como sucesión de Fibonacci: 1, 2, 3, 5, 8, 13, 21, 34, 55, . . .. El problema de

50

determinar el número de parejas de conejos al final del primer año es simplemente el de calcular F12,que es igual a 377.

EJEMPLO 4.5El Modelo de Reproducción de Conejos y las Cadenas de ceros y unos que no enen dos unos seguidos.

Veremos como este problema del número de parejas de conejos al final del enésimomes puede ponerseen correspondencia con el problema de las cadenas de ceros y unos de longitud n que no enen dos unosseguidos.

Suponga una cadena de ceros y unos ene la propiedad de no tener dos unos consecu vos. Cada 1 en lacadena representará el nacimiento de una pareja; cada pareja de conejos, se iden ficará con el úl mo1 en la cadena; los demás unos de la cadena que aparecen antes, representarán el árbol genealógicode la pareja que representa la cadena. La cadena de sólo ceros estará asociada a la pareja original. Larestricción de que las cadenas no tengan dos 1 consecu vos es justamente debido a que en el modelocada pareja sólo puede comenzar a procrear al finalizar el segundo mes.Por ejemplo, la cadena 00101001 es el documento de iden dad de la pareja de conejos que nació alfinalizar el octavomes, cuyos padres nacieron en el quintomes; sus abuelos nacieron al finalizar el tercermes y sus bisabuelos la pareja de conejos original.

Es evidente que esta correspondencia es biunívoca; a cada cadena de ceros y unos con la propiedad deno tener dos unos seguidos, le corresponde una sola pareja de conejos y a cada pareja de conejos, lecorresponde una sola cadena.

Podemos ahora, apoyados en la correspondencia anterior, proponernos contar el número de parejasde conejos, mediante el conteo de las cadenas de ceros y unos con la propiedad de no tener dos unosseguidos. Sólo con el propósito de ilustrar nos restringiremos al caso de cadenas de longitud 6, el casogeneral puede ser analizadode forma completamente análoga. El problemaes así el de contar las cadenasde longitud 6 de ceros y unos con la propiedad de no tener dos unos seguidos.

Obsérvese primero que para que no hayan dos unos seguidos se requiere que el número de unos seamenor o igual a tres, de lo contrario no se dispondrá de suficientes ceros para separarlos. Podemosentonces clasificar nuestras cadenas según el número de unos que posea: Cadenas que no enen unos;los 6 son ceros y los espacios disponibles para colocar los unos, son 7. Se ene entonces C07 en este caso.Cadenas que enen exactamente un uno. Obviamente la cadena debe tener 5 ceros, lo que permitedisponer de 6 espacios para colocar el uno, en este caso el número de cadenas debe ser C16 . He aquí lalista de todas estas cadenas: 100000, 010000, 001000, 000100, 000010, 000001. Cadenas que enenexactamente dos unos. En este caso deben haber cuatro ceros, lo que permite disponer de 5 espaciospara colocar los 2 unos. El total deben ser C25 cadenas, que son los siguientes: 101000, 100100, 100010,100001,010100,010010,010001,001010,001001,000101. Cadenas que enen tres unos. Por supuestola cadena ene tres ceros y los espacios disponibles para colocar los tres unos, son 4; por lo tantotendremos C34 cadenas; estas son: 101010, 101001, 100101, 010101.

51

El total de cadenas con la propiedad será entonces la suma: 1+6+10+4 = 21, número que correspondea F6 en la sucesión de Fibonacci.

4.7. PRINCIPIO DE CASILLAS

El principio de casillas es posiblemente uno de los teoremas más evidentes, y cualquiera podría pensarqueno eneu lidad algunauna aseveración tan evidente, sin embargo, en los problemas que se planteanen este capítulo se puede apreciar lo poco evidente que resultan muchos problemas cuya clave radicaen el principio de casillas.

EJEMPLO 4.6Una recta no puede cortar internamente a los tres lados de un triángulo simultáneamente.

En este caso hay que crear tanto las cajas como los objetos; digamos que se traza la recta L,10 la cualgenera dos semiplanos, éstos serán las cajas; por otra parte, los vér ces del triángulo serán los objetos,que son en tres en total. Por el principio de casillas, hay un semiplano que ene al menos dos vér ces,por lo tanto, la recta L no corta al lado definido por esos vér ces.

4.8. PROBLEMAS

1. ¿Cuántos enteros entre 1 y 1000, incluyéndolos, no son divisibles entre 2, 3 ó 5?

2. ¿Cuántos números naturales menores o iguales que 10000 son múl plos de 4, 5 ó 7? ¿Cuántosson múl plos de 4, 10 ó 14?

3. En cierta escuela hay 100 alumnos. De ellos, 50 saben inglés, 30 saben alemán y 30 saben francés.Además, 10 saben inglés y francés, 14 saben francés y alemán, 11 saben inglés y alemán, y 6 sabenlos tres idiomas. Determinar cuántos alumnos no saben ninguno de los tres idiomas.

4. Un grupo de 102 estudiantes se examinan enMatemá cas, Sociales y Lenguaje. De entre ellos, 92pasaronMatemá cas,75 Sociales y63 Lenguaje,65pasaronMatemá cas y Sociales,54Matemá casy Lenguaje y 48 Sociales y Lenguaje. ¿Cuántos estudiantes pasaron las tres materias?

5. ¿Cuántos números del 1 al1000000no son ni cuadrados perfectos, ni cubos perfectos, ni potenciascuartas perfectas?

6. En un grupo de 100 indios hay 40 que hablan hindi, 40 que hablan bengalí y 20 que hablan penjabi.Hay 20 que hablan hindi y bengalí, y 5 que hablan hindi y penjabi. Hay 31 que hablan al menos dosde las tres lenguas y 33 que no hablan ninguna de ellas. ¿Cuántos hablan las tres lenguas?

7. ¿Cuántos númerosmenores que 400 enen la propiedad de no tener divisor alguno en el conjunto6, 10, 15?

10Como se busca cortar internamente a los lados del triángulo, suponemos que L no coincide con ninguno de los lados nipasa por algún vér ce.

52

8. ¿Cuántos números naturales menores que 1000 enen la propiedad de ser divisibles por 12, perono divisibles ni por 5, ni por 7?

9. De todas las combinaciones posibles de 5 elementos del conjunto 1, 2, . . . , 10 ¿Cuántas noincluyen ni el número 8, ni el número 9?

10. Determine el número de desórdenes posibles del conjunto 1, 2, 3, 4, 5.

11. Si se asume el orden natural en el conjunto 1, 2, 3, 4, 5, ¿cuántas permutaciones dejan fijos ensu posición exactamente a dos de los cinco números?

12. Considere los números naturalesmenores o iguales a100 en sudescomposición en factores primos.¿En cuántos de ellos no hay un factor primo repe do?

13. Un año es bisiesto cuando:

) Es múl plo de 4 pero no de 100, o

) Es múl plo de 400.

Por ejemplo, 1600 y 1924 fueron años bisiestos, mientras que 2200 no lo será. Encuentre elnúmero total de años bisiestos entre los años 1000 y 3000.

14. ¿Cuántas permutaciones de 1234 son tales que el 1 no está en la primera posición, el 2 no está enla segunda posición, el 3 no está en la tercera posición, y el 4 no está en la cuarta posición?

15. En una oficina hay 10 empleados. Cada uno es especialista en una labor dis nta a la de los demás.Para no aburrirse, les gusta intercambiar sus puestos; sin embargo, el buen funcionamiento de laoficina exige que en cadamomento haya exactamente 4 empleados trabajando en su especialidad.¿Cuántas distribuciones de los puestos se pueden hacer bajo estas condiciones?

16. Se sabeque la clave de acceso deuna comutadora es unapermutacióndeCLAVE2654. Sin embargo,como el dueño sufre de dislexia, programó en su compuradora la siguiente regla: Para conseguiracceso, es suficiente introducir5 caracteres en la posición correcta, los caracteres restantes puedenestar en la posición correcta o no. ¿De cuántas formas puede acceder a la computadora?

17. Ana quiere comprar 15 dulces, los sabores disponibles son: coco, vainilla, fresa y canela. Como asu mamá le gustan los dulces de coco, piensa llevar al menos 5 de este sabor; además, nunca haprobado los dulces de canela, por lo que llevará a lo sumo 4 de este sabor. ¿De cuántas formaspuede Ana comprar los dulces?

18. Para comprar en una carnicería cada cliente toma un número (los números van saliendo ordenados1, 2, . . . , n), pero el carnicero estaba de mal humor un día y decidió atender a las n personas queesperaban su turno de una manera muy extraña. ¿De cuántas formas puede atenderlos si los queenen número impar serán atendidos en su turno, mientras que los que enen número par no?

19. En un grupo de 102 estudiantes se examinan en Matemá cas, Sociales y Lenguajes. De entreellos, 92 pasaron Matemá cas, 75 Sociales y 63 Lenguaje, 65 pasaron Matemá cas y Sociales, 54Matemá cas y Lenguaje, y 48 Sociales y Lenguaje. ¿Cuántos estudiantes pasaron las tresmaterias?

53

20. Para rendir un examen, 4 alumnos se sientan en una fila de 10 asientos. ¿De cuántas maneraspueden ubicarse, si no puede haber alumnos sentados con guamente?

21. Siete libros diferentes, tres de Historia, dos de Matemá ca y dos de Química, son colocados en elestante de una biblioteca. ¿En cuántas de las posibles formas de ubicarlos no aparecerán todos loslibros de una misma materia?

22. ¿De cuántas formas se puede colocar tres x , tres y y tres z de modo que no aparezcan la mismaletra tres veces consecu vas?

23. En cierto ecosistema hay 18 especies de animales. Cada especie depredadora caza 2 especiesdiferentes. A su vez cada especie no depredadora es perseguida por 3 especies diferentes. Además,se sabeque toda especie es perseguida odepredadora, y ningunade las dos cosas a la vez. ¿Cuántasespecies depredadoras hay?

24. Consideremos una cuadricula de n × n ¿cuántos cuadrados tales que apoyen alguno de sus ladosen el borde inferior o el borde izquierdo de de la cuadricula?

25. Consideremos una cuadricula de 8 × 10 casillas. ¿Cuántos rectángulos pueden marcarse en ellaque tengan por lo menos uno de sus lados apoyado en algún borde?

26. ¿Cuántas soluciones enteras ene la ecuación x1 + x2 + x3 = 28 si las variables están sujetas a lacondición: 3 ≤ x1 ≤ 9, 0 ≤ x2 ≤ 8, 7 ≤ x3 ≤ 17?

27. ¿Cuántas soluciones enteras ene la ecuación x1+ x2+ x3+ x4 = 20 si las variables están sujetasa la condición x1 ≤ 6, x2 ≤ 7, x3 ≤ 8, x4 ≤ 9?

28. Hallar el número de soluciones enteras de la ecuación x1 + x2 + x3 + x4 + x5 + x6 = 20 quesa sfacen las condiciones: 1 ≤ x1 ≤ 6, 1 ≤ x2 ≤ 7, 3 ≤ x3 ≤ 9, 4 ≤ x4 ≤ 11.

29. ¿De cuántas formas se pueden distribuir 10 premios dis ntos entre 4 estudiantes de modo queexactamente dos estudiantes no reciban ninguno? ¿De cuántas formas puede hacerse esto demodo que al menos dos estudiantes no reciban premio?

30. Demuestre mediante la definición de coeficiente mul nomial que:(n

n1, n2, . . . , nm

)=

m∑i=1

(n − 1

n1, . . . , ni − 1, ni + 1, . . . , nm

)(1)

31. Demuestre que ∑(n

n1, n2, · · · , nm

)= mn

donde el sumatorio contempla todas las soluciones enteras no nega vas de n1 + n2 + · · · nm = n

32. Sea k, n, r ∈ N. Muestre que el número de soluciones enteras de la ecuación

x1 + x2 + x3 + · · ·+ xn = r

54

tal que 0 ≤ xi ≤ k para cada i = 1, 2, . . . , n está dado porn∑i=0

(−1)i(n

i

)(r − (k + 1)i + n − 1

n − 1

)33. Determinar el número de regiones determinadas por n rectas en el plano.

34. Determine el número de cadenas de ceros y unos que enen la propiedad de no tener dos unosconsecu vos.

35. Supongamos que se desea cubrir un rectángulo de dimensión n×2 con rectángulos de 2×1. ¿Decuántas formas es posible hacerlo?

36. ¿De cuántas formas es posible descomponer un número natural como suma sólo de los naturales1 y 2?

37. Llamamos triangulación de un polígono a su descomposición en triángulos cuyos lados son ya seandiagonales o lados del polígono, de forma tal que el polígono queda completamente cubierto, sinsuperposiciones.

a) Demuestre que, independiente de la triangulación, el número de triángulos u lizados en ladescomposición es el mismo e igual a n − 2.

b) Demuestre que el número de diagonales u lizadas en cualquier triangulación es siempre n−3.c) Determinar el número de posibles triangulaciones de un polígono de n lados.

38. Considere un triángulo equilátero cuyo lado ene longitud n. Se triángula trazando rectas paralelasa los lados que cortan segmentos de 1 a los lados del triángulo; se forman triángulos equiláterosde diferentes longitudes de lado. Determine el número total de tales triángulos.

39. Se dispone de un número ilimitado de monedas de las denominaciones de 1,5,10 y 25 centavosde dólar. ¿De cuántas formas se puede formar un total de 30 centavos? Determine una relación derecurrencia que permita calcular el número de posibilidades de cambiar una can dad cualquierade dinero.

40. Sean C un conjunto con 2n números reales, n ≥ 1 y an el número de comparaciones que debenefectuarse entre los elementos de C para determinar el máximo y el mínimo de C. Encontrar unarelación de recurrencia para calcular an y resolverla.

41. Tenemos cinco puntos con coordenadas enteras en el plano cartesiano. Probar que si sacamos lospuntos medios para cada par de estos puntos, exis rá un punto medio cuyas coordenadas serántambién enteras.

42. Una prueba de concurso posee diez preguntas de selección múl ple, con cinco alterna vas cadauna. ¿Cuál es el número mínimo de candidatos que deberían hacer el examen para garan zar quepor lo menos dos de ellos tendrán las mismas respuestas para todas las preguntas?

43. Dados 6 puntos sobre una circunferencia, coloreamos de azul o verde todos los segmentos queellos determinan. Demuestre que entre todos los triángulos que quedan formados, hay por lomenos uno cromá co, es decir, un triángulo con sus tres lados del mismo color.

55

44. Demuestre que si con 2 colores se pintan las diagonales de un pentágono regular, siempre hay unvér ce del que salen dos diagonales del mismo color.

45. Para una reunión cien fica se han contratado5 traductores, quedeberán cubrir6 lenguas diferentes.Si cada una de éstas requiere el empleo de 3 traductores, demostrar que alguno de los intérpretesdeberá traducir por lo menos 4 idiomas.

46. En un cajón hay calce nes negros, rojos, azules y blancos. ¿Cuál es el menor número de calce nesque hay que sacar para estar seguros de que hay al menos dos del mismo color?

47. En un estadio hay diez mil personas. Demostrar que hay al menos un grupo de 28 personas quenacieron el mismo día.

48. ¿Cuál es el mayor número de reyes que pueden ser colocados en un tablero de ajedrez de maneraque ninguno dé jaque a ningún otro?

49. A un estadio de fútbol han asis do 37000 espectadores. ¿Cuántos de ellos, comomínimo, cumplenaños el mismo día?

50. Hay 100 personas sentadas en una mesa circular a distancia constante entre sí y al menos 51 deellas son mujeres. Verificar que hay al menos 2 mujeres sentados en posiciones diametralmenteopuestas.

51. Considere los primos 2, 3, 5. El conjuntoA está formado por los números naturales que se generanmul plicando dis ntas potencias de estos primos, es decir, los números de la forma

2α3β5γ

con α, β, γ enteros no nega vos. Demuestre que de cualquier escogitación de 9 números de Asiempre hay 2 que al mul plicarlos generan un cuadrado perfecto.

52. Suponga que los números del 1 al 10 están ubicados en algún orden sobre una circunferencia.Prueba que alguna suma de tres números consecu vos suma 17.

53. En un planeta llamado Ω (omega), más de la mitad de la superficie es erra firme. Probar que enΩ se podría excavar un túnel recto a través del centro del planeta, comenzando y finalizando enerra firme.

54. ¿Cuántas veces, como mínimo, debe lanzarse un par de dados para asegurarse que el puntajeobtenido (la suma de los dados) se repita?

55. Sean a, b, c, d enteros, demuestre que (a− b)(b− c)(c − d)(d − a)(a− c)(b− d) es múl plode 12.

56. Una prueba de ap tud posee 10 preguntas de selección múl ple, con cinco alterna vas cada una.¿Cuál debe ser elmínimonúmero de alumnos que debendar la prueba (sin dejar respuestas vacías)para el cual podamos garan zar que por lo menos dos de ellos tendrán exactamente las mismasrespuestas para todas las preguntas?

56

57. En una caja hay 10 libros en francés, 20 en castellano, 8 en alemán, 15 en ruso y 25 en italiano.¿Cuantos debo sacar para estar seguro de que tengo 12 en un mismo idioma?

58. En un bar hay 95mesas y un total de 465 sillas. ¿Podemos asegurar que hay unamesa con 6 sillas?

59. Se eneun conjunto dediez números naturales. Demostrar quehay almenos unpar cuya diferenciaes múl plo de 9.

60. Demuestre que si del subconjunto de números naturales 1, 2, . . . , 10 extraemos seis números,con seguridad habrá dos que suman 11.

61. De los números1, 2, . . . , 100 se toman51. Demuestre quede estos, hay una pareja que sonprimosrela vos, y que hay otra pareja tal que uno divide al otro y el cociente es potencia de 2.

62. Sean a1, a2, . . . , a100 y b1, b2, . . . , b100 dos permutaciones de 1, 2, 3, . . . , 100. Demuestra que,entre los productos

a1b1, . . . , aibj , . . . , a100b100

hay dos con el mismo residuo al dividirse entre 100.

63. Los números 1, 2, . . . , 9 se dividen en tres grupos. Probar que el producto de los números en unode dichos grupos, sean cual sean estos, siempre debe ser mayor de 71.

64. En un cubo de lado 10 se colocan 999 puntos. ¿Es posible encontrar siempre un cubo de lado 1dentro del cubo de lado 10 que no contenga alguno de los puntos?

65. Con los vér ces de una cuadrícula de 6×9 se forman24 triángulos. Demuestre que hay 2 triángulosque enen un vér ce en común.

66. Las entradas de una matriz 3× 3 son los números 0, 1,−1. Probar que entre las ocho sumas quese ob enen por filas, columnas y diagonales, hay dos iguales.

67. Se enen los números 1, 2, . . . , 2n escritos en una pizarra. Se tachan n1 de ellos. Probar que entrelos números que quedaron sin tachar en la pizarra, hay almenos dos de ellos que son consecu vos.

68. En una pizarra se escriben los números 1, 2, . . . , 2n, probar que si se eligen aleatoriamente n+ 1números de entre ellos, entonces entre los elegidos habrá un par de modo que uno divide al otro.

69. Dados 27 números impares posi vos menores que 100, demostrar que hay al menos dos de elloscuya suma es 102.

70. 17 personas se comunican por correo, enviando cada persona una carta a cada una de las demás.En las cartas sólo son discu das tres temá cas dis ntas. Las cartas enviadas mutuamente entredos personas tratan ambas a cerca de una sola de esas temá cas, dado que una persona envíauna carta y la persona a la que va dirigida le responde. Pruebe que hay un grupo de al menos 3personas tales que en todas las cartas que se enviaron entre sí discu eron a cerca de la mismatemá ca.

71. Algunos de los cuadritos de una cuadricula de 3 × 7 se pintan de negro y los otros se dejan enblanco. Probar que forzosamente las líneas de la cuadrícula forman un rectángulo en cuyas cuatroesquinas los cuadraditos enen el mismo color (los cuatro blancos o los cuatro negros).

57

72. Dado un cuadrado de diagonal 3, se marcan al azar 10 puntos. Demostrar que siempre podemosencontrar al menos dos puntos que están a una distancia no mayor a 1.

73. En un triángulo de área 4 se colocan nueve puntos. Muestre que hay tres de ellos que forman untriángulo de área menor o igual a 1.

74. Demuestre queun triángulo equilátero de lado1nopuede ser cubierto totalmente condos triángulosequiláteros de lados menores que 1.

75. Un disco cerrado de radio 1cm con ene 7 puntos tales que todas las distancias entre dos de ellosson mayores o iguales que 1, pruebe que uno de los 7 puntos es el centro del disco.

76. Nos damosun conjuntoX de10números naturales dis ntos comprendidos entre10 y99. Demostrarquehay dos subconjuntos dis ntos deX tales que la sumade los elementos de los dos subconjuntosdan el mismo resultado.

77. En un segmento I de longitud 10, inicialmente blanco, se han marcado 10 segmentos disjuntoscon color rojo. Si no hay dos puntos en I a distancia 1 y ambos de color rojo, probar que la sumade las longitudes de los intervalos es a lo sumo 5.

78. Se dispone de 100 tarjetas, numeradas del 100 al 199. El valor de cada tarjeta es la suma de losdígitos que aparecen en ella. ¿Cuál es el numero mínimo de tarjetas que hay que extraer paraasegurar que haya tres con el mismo valor?

79. Demostrar que enunafiesta siemprehaydos personas que conocen almismonúmerodepersonas.

80. Comprobar que en una reunión de 6 personas siempre pasa que 3 de ellas se conocen entre sí obien 3 de ellas no se conocen entre sí.

81. En una ceremonia de premiación, n personas se saludaron entre sí estrechándose las manos.Prueba que durante la ceremonia hubo siempre dos personas que estrecharon el mismo númerode manos.

82. En una reunión hay 201 personas de cinco nacionalidades diferentes. Se sabe que en cada grupo deseis, al menos dos enen la misma edad. Demuestra que hay al menos cinco personas del mismopaís, de la misma edad y del mismo sexo.

83. Demuestre las siguientes iden dades albegraicas:

a)n∑i=1

i = 1 + 2 + 3 + · · ·+ n =n(n + 1)

2

b)n∑i=1

i(i + 1) = 1 · 2 + 2 · 3 + 3 · 4 + · · ·+ n(n + 1) =n(n + 1)(n + 2)

3

58

c)

n∑i=1

i(i+1)(i+2) = 1·2·3+2·3·4+3·4·5+· · ·+n(n+1)(n+2) =n(n + 1)(n + 2)(n + 3)

4

d) Exprese una iden dad algebraica que generalice las iden dades anteriores y demuéstrela.

84. U lizando las iden dades anteriores, demuestre

a)n∑i=1

i2 = 12 + 22 + 32 + · · ·+ n2 =n(n + 1)(2n + 1)

6

b)n∑i=1

i3 = 13 + 23 + 33 + · · ·+ n3 =n2(n + 1)2

4=

(n∑i=1

i

)2c) Encuentre una iden dad algebraica para la suma de las potencias cuartas de 1 a n.

85. Sea S = 1, 2, 3, . . . , n + 1 en donde n ≥ 2, y sea T = (x, y , z) ∈ S3|x < z, y < z. Alcontar |T | de dos modos dis ntos, muestre que:

n∑k=1

k2 = |T | =(n + 1

2

)+ 2

(n + 1

3

)

86. Demuestre que(3n)!

2n3nes entero para todo natural n.

87. Sea A = 1, 2, 3, . . . , n, donde n ∈ N

a) Para k ∈ A, muestre que el número de subconjuntos de A en los que k es el número mayor, esigual a 2k−1.

b) Muestre a par r de lo anterior que:

n−1∑i=0

2i = 2n − 1. (2)

Lectura Complementaria: EderAlexander Jacobo.Material deAutoformaciónDocenteparaMatemá ca9° grado. Lección 8. Desarrollo binomial y Mul binomial. MINED

59